Sei sulla pagina 1di 34

6/23/2014 Evernote shared notebook:

https://www.evernote.com/pub/crazyphoton/interviewprep#st=p&n=7a5f4d76-f67a-42ea-8484-e7aaa3d53c47 1/34
Search
interview prep
crazyphoton has shared a
notebook with you.
Join Notebook
interview prep
Personal Issues
4/26/14 Q. Many engineers have come f or interview in thisboard. I
am sure that it must be happening in other boards also. Why so
manyengineers are coming to the civil services, especially f rom top
Economics Issues
4/24/14 Nayar Committee Report But, more
importantly, as the report itself highlights, public
sector banks in India are caught in a triple whammy:
IT Issues
4/24/14 Internet of Things IDC also f orecasts that by 2020 Internet
of Things (IoT) will represent almost 10 per cent of the data
generated in India. IoT ref ers to everyday objects equipped with
Governance and Social Issues
4/24/14 1. Build conf idence in bureaucracy 2. Welcome innovative
ideas & babus to be given f reedom to work 3. Education, Health,
Water. Energy & roads will be priority 4. Transparency in the
IIM, IIT, Education, Blogging, Bharatpur, Hong Kong
2/18/13 Education Q. What is your view about
allowing f oreign universities to establish their
institutions in India? welcome step - economy needs
Rajasthan, Jaipur, Kanpur, Lucknow, Hyderabad
1/19/13 Dara Singh and Sorahbuddin encounter cases Solar energy
in Rajasthan Rajasthan is one of the leading states of India in the
f ield of solar energy. The total photovoltaic capacity has passed 500
Police Issues
1/10/13 Q. What are the problems of Law and Order in Rajasthan
and what are their solutions? 4. Why there are issues in Police
Administration in India and how will you solve them?? Kerala
Misc
11/12/12 International Af f airs Q. Do you think India is a sof t state?
No. I don't agree with the concept of a sof t state. Every state today
def ends its national interests and if it doesn't def end one, its beca...
1-8 of 8 notes
Econo
Nayar
Committee
Report
But, more
importantly, as
the report itself
highlights, public
sector banks in
India are caught
in a triple
whammy:
deteriorating
asset quality,
massive capital
requirements
stemming from
Basel III norms,
and a complete
lack of strategic
manoeuvrability,
given their
current
governance and
management
structures. While
the report
addresses issues
relating to the
governance of
private banks as
well, its most
significant
recommendations
could put the
entire public
sector banking
system on to a
interview prep INFO TOOLS
Created: Apr 24, 2014
Economics Issues
Thursday, April 24 2014, 7:28 PM
Nayar Committee Report
But, more importantly, as the report itself highlights, public sector banks in India are caught in a triple whammy:
deteriorating asset quality, massive capital requirements stemming from Basel III norms, and a complete lack of strategic
manoeuvrability, given their current governance and management structures. While the report addresses issues relating to
the governance of private banks as well, its most significant recommendations could put the entire public sector banking
system on to a new, sustainable performance and risk management trajectory.
The committee shares the widespread perception that excessive and misdirected government control is at the heart of the
problem. Consequently, it believes that reducing the government equity stake to minority levels and then empowering
boards and managements to function within the performance and accountability frameworks of typical corporate
organisations is the best way for these banks to get themselves out of the three-way trap
Nayak Committee Report
What it would like is for the government to distance itself from several bank regulatory functions it discharges. To do this,
it would like the repeal of the Bank Nationalisation Act of 1970 and 1980, SBI Act and that relating to its subsidiaries; all
banks should be incorporated under the Companies Act.
The governments holdings should be transferred to a Bank Investment Company (BIC) along with its regulatory
functions. The governments holdings in PSBs should come down to less than 50 per cent, but it can and will remain the
dominant partner. But its regulatory functions will now pass to the BIC. At this point, one might think that the Nayak
Committee is talking about privatisation. Its concerns, however, seem to lie elsewhere.
The key lies in the appointment of boards and senior officials in banks. Right now the finance ministry plays big brother
with most appointments of bank chiefs. Under the new dispensation, a Bank Boards Bureau would deal with senior level
appointments till the BIC is formed. Eventually, at the end of a three year process, however, the responsibilities would rest
with the boards themselves.
It is interesting that among other things, the committee should list the RTI Act as a deterrent. Look across the spectrum
of services Indian citizens are offered and you will find a similar sentiment expressed by bureaucrats and even prestigious
higher educational institutions.
. A core recommendation is to unshackle banks from government control and finance ministrys interference, by reducing
government holding to under 50% and bringing banks under the sole regulation of RBI. It suggests that when government
shareholding is brought below 50%, the rest should be transferred to a bank investment company (BIC).
The committee recommends the creation of a category of authorised bank investors (ABIs) who can invest in banks
without prior approval.
It recommends that these privatised PSU banks be removed from dual regulation of the finance ministry and RBI and
brought solely under RBI.
Q. Give me few problems ailing our economy and solutions to address them ?
Low growth - manufacturing as well as services.
Stalled projects.
Multiple clearances, bureaucratic red tape.
Indias rank in the World Banks Ease of doing business index fell from 116 out of 189 countries in 2006 to 134 in 2013
6/23/2014 Evernote shared notebook:
https://www.evernote.com/pub/crazyphoton/interviewprep#st=p&n=7a5f4d76-f67a-42ea-8484-e7aaa3d53c47 2/34
Indias rank in the World Banks Ease of doing business index fell from 116 out of 189 countries in 2006 to 134 in 2013
clear evidence of stalled reforms. The new government needs to reverse this trend and improve the environment for doing
business.
Unskilled workers / low human capital development.
Rope in private sector, allow for profit.
Supply side constraints in infrastructure - roads, power plants.
Road sector - fresh bids, independent regulator to oversee performance, CAG audit.
Power sector - free up coal mining, pass through mechanism, discom reforms, CAG audit.
Ports sector - free up port management and construction.
Railways sector - More PPP, independent regulator and auditor.
Uncertain policy environment.
Increase transparency, reduce discretionary decision making.
in recent years the lack of fiscal discipline has been costly for the Indian economy, as excessive demand arising from
large deficits translated into stubbornly high inflation and was partly responsible for large current-account deficits. Fiscal
discipline should be a priority, not an afterthought.
improving the quantity and quality of education and healthcare through partnerships with the private and non-profit sector
and researchers is essential to sustain growth beyond the next five years.
There are currently three ministries in the energy sectorpower, petroleum and natural gas, and renewable energy. It
makes sense to fuse them into a single ministry. The recent problem of power plants being built without proper coal
supplies could have been avoided if policy had been handled by one minister. There is no need for a ministry of
information and broadcasting, as the former minister in charge of the portfolio, Manish Tewari, reportedly admitted last
week. Is there a need for a separate ministry of urban poverty alleviation? A ministry of culture? A ministry of heavy
industries? And a ministry for at micro, small and medium enterprises? What about a ministry of pensions?
Foodgrain stocks, as on April 1, 2014, were estimated at 38 million tonnes, way above the minimum buffer requirement.
To the extent that there is a speculative increase in foodgrain prices, the government should aggressively sell its stocks in
the market.
From all counts, there would be an avalanche of capital inflows in the immediate ensuing period; this would swell domestic
liquidity and generate inflation, unless immediate countervailing measures are taken.
The opportunity should be used to build up the forex reserves. Also, there should be a tilt towards longer-term flows rather
than short-term flows and more foreign direct investment rather than portfolio investment.
The share of manufacturing in GDP declined from 14.8 per cent in 2011-12 to 14.1 per cent in 2012-13 and to 12.7 per
cent in 2013-14, implying that this sector is steadily losing its relevance in the economy, and that the real economy is
weakening.
Poor growth of the manufacturing sector has resulted in joblessness. This can be gleaned from the performance of two
sub-segments of manufacturing capital goods and consumer durables. Both segments are highly intensive in the use of
components and ancillaries, and thereby encourage growth of large number of small and medium scale industries as they
grow.
Any crisis in these two sub-segments affects the supporting SMEs and gives rise to unemployment.
Growth in capital goods and consumer goods sectors suffers due to lack of investment and consumption demand. In the
case of basic and intermediate goods, however, growth is hampered by lack of governance and excessive regulatory
hurdles and interventions.
Govt should follow cluster approach and give investment incentives.
Q. What are 2nd generation reforms.
Manufacturing reforms
Labor reforms.
Land acquisition.
Multiple clearances and single window.
Governance reforms
Transparency and rule of law.
Rationalization of subsidies.
6/23/2014 Evernote shared notebook:
https://www.evernote.com/pub/crazyphoton/interviewprep#st=p&n=7a5f4d76-f67a-42ea-8484-e7aaa3d53c47 3/34
Rationalization of subsidies.
Restructuring of government schemes.
Trade reforms
GST and unified market.
Financial reforms
Financial sector liberalization and more competition.
FSLRC recommendations.
Q. Whether subsidies good or bad?
In a poor country like India, certain subsidies are essential. Eg. talk about PDS and Himanshu's findings.
What we need is to plug leakages in the subsidies.
UID was a good initiative, but after SC's decision, its fate is undecided.
Still we can reform subsidy regime by use of IT and mobile. eg. PDS.
Also we can create NREGA like architecture where demand is generated from the bottom and is backed by legal rights.
We also need effective grievance redressal mechanisms.
Social audits.
The Indian media rails against the governments fiscal irresponsibility in introducing food subsidies, but the government
spends far more on subsidies for the rich, including 1% on electricity and 1% on diesel and petroleum products, which
benefit the middle classes.
Q. What should we do to promote manufacturing sector?
Cut red tape, reduce number of clearances, single window mechanism. Transparency, reduce discretions.
Reduce compliance costs.
Labor laws.
Develop infrastructure.
Skill development.
Land acquisitions.
Friendly policies.
Amartya Sen
Nobel Prize
Sen got the noble prize for his work in - social choice (Impossibility theorem), distribution, poverty (Capabilities based
approach).
Sen's Capabilities Based Approach
Functionings are states of being and doing which measure an individual's well being such as being well-nourished, having
shelter.
Capability refers to the set of valuable functionings that a person has effective access to. Capability represents the
effective freedom of an individual to choose between different functioning combinations.
Freedoms = capabilities. Unfreedom = deprivation of capabilities.
Poverty is a serious deprivation of certain basic capabilities to live a good life, and development is understood as
capability expansion.
Income approach to poverty is not enough: Defining poverty as simply income below an accepted level is not enough as
there can be variations in converting this income into various capabilities. These variations may arise due to personal,
environmental, social factors. Hence the need to go beyond income in poverty measurement.
Utility approach to poverty is not enough: People can internalize the harshness of their circumstances so that they do not
desire what they can never expect to achieve. This is the phenomenon of adaptive preferences in which people who are
objectively very sick may, for example, still declare, and believe, that their health is fine.
Resource based approach to poverty is not enough: Because it only focuses on what goods a person has, not what he
can do with those goods.
Q. sen's policies have become irrelevant now.
Not irrelevant, but the new government may be less guided by them.
In fact rights based approach is in alignment with the new thought in administration that the citizen must be empowered
6/23/2014 Evernote shared notebook:
https://www.evernote.com/pub/crazyphoton/interviewprep#st=p&n=7a5f4d76-f67a-42ea-8484-e7aaa3d53c47 4/34
In fact rights based approach is in alignment with the new thought in administration that the citizen must be empowered
and government held accountable. RTI, NREGA are examples of success.
What we need is to cut down on wasteful expenditure and red tape, opacity. We need to increase transparency and rule of
law.
Q. Compare n contrast bhagwati sen model
Bhagwati model
He talks of 2 track reforms. Track 1 reforms are reforms aimed at boosting GDP growth and industrialization. It involves
building infrastructure, increasing transparency, incentives to industry etc.
Track 2 reforms which will follow track 1 are increased state spending on education and health.
Track 2 can follow track 1 because: a) Track 1 will help lift people out of poverty via additional jobs and incomes. b) Track
1 will also generate revenues for the state to spend on track 2.
Sen model
Public provision of health and education is a must because if labor are not educated and healthy, they cannot have high
productivity and growth will not increase. There is not a single example in the world where development has been possible
without investing in education and health.
Sen is not against growth, but he says we must also focus on what this growth does to people's lives.
Q Compare n contrast bihar n gujarat model. Which one do we need?
Bihar model
State actively engaged in promoting education, food and health facilities to citizens.
Gujarat model
State creates a business friendly environment. Private investment led model.
What we need?
We need a synthesis. India is too big a country where conditions in one part are completely different from another part.
For example, in a state like Bihar where poverty and illiteracy are high, government incentives are needed for people to
study up to class X. But such a thing is unnecessary in Gujarat where already literacy rates are high. If people are
illiterate, then just focusing on private investment will not work to promote an inclusive growth.
We need to focus on what kind of growth we are having. We need a growth which creates jobs.
Q. what happened in recent wto?
Trade facilitation: A deal was reached. Red tape and delays have to be cut in allowing international trade.
AoA: India got a 4 year peace clause after which the fate of subsidies will be decided.
RBI Monetary Policy Review
Recent Policy Action (April 1)
Increased the liquidity provided under 7-day and 14-day term repos from 0.5% of NDTL to 0.75% in line with Urijit Patel
committee recommendations, and decrease the liquidity provided under the overnight repos from 0.5% to 0.25%. The
primary objective is to improve the transmission of policy impulses across the interest rate spectrum.
Policy stance
Kept repo rate under LAF unchanged at 8%.
The policy stance is firmly focused on keeping the economy on a disinflationary glide path that is intended to hit 8 per
cent CPI inflation by January 2015 and 6 per cent by January 2016.
Furthermore, if inflation continues along the intended glide path, further policy tightening in the near term is not anticipated
at this juncture.
Macro Forecast
GDP growth
Itis expected to pick up from below 5% in 2013-14 to 5 - 6% in 2014-15 though with downside risks to the central estimate
of 5.5 per cent.
Positive factors
Progress on the implementation of stalled projects already cleared.
Stronger anticipated export growth as the world economy picks up.
6/23/2014 Evernote shared notebook:
https://www.evernote.com/pub/crazyphoton/interviewprep#st=p&n=7a5f4d76-f67a-42ea-8484-e7aaa3d53c47 5/34
Stronger anticipated export growth as the world economy picks up.
CAD
CAD is expected to be about 2.0% of GDP for 2013-14.
Inflation
Forecast of 8 per cent CPI inflation by January 2015.
Risk factors
Vegetable prices have entered their seasonal trough and further softening is unlikely.
El Nino and uncertain rainfall.
Uncertainty on the setting of MSPs.
Uncertainty in fiscal policy.
Geo-political developments and their impact on international commodity prices.
There will also be a downward statistical pull on CPI inflation later this year, due to base effects from high inflation during
June-November 2013.
Urijit Patel Committee Recommendations
Some recommendations of Dr. Urjit R. Patel Committee report have been implemented including
Adoption of the new CPI (combined) as key measure of inflation.
Explicit recognition of the glide path for disinflation (8% for Jan 1015, 6% for Jan 2016, 4% thereafter).
Transition to a bi-monthly monetary policy cycle.
Progressive reduction in access to overnight liquidity under the LAF at the repo rate and a corresponding increase in
access to liquidity through term repos, and introduction of longer term repos.
Bimal Jalan Committee Recmmendations
RBI will work to give licenses more regularly, that is virtually on-tap.
It will also set out categories of differentiated bank licenses that will allow a wider pool of entrants into banking.
Market Development Efforts by RBI
Inflation bonds
To expand investor demand, design changes improving their attractiveness to the general public are being worked out.
Corporate bonds
Banks will be allowed to offer partial credit enhancements to them.
Re-repo of g-secs
The feasibility of limited re-repo/re-hypothecation of repoed government securities is being explored.The idea of
rehypothecation typically works not so much with term repos but reverse repos, with somebody sort of lent money and
suddenly feels that they have a squeeze on their reserve they want to relend to somebody else, rehypothecation allows
that. So as you move to term reverse repos this will be a helpful thing, and we are exploring it, we think we can do it
without much risk.
FIIs
FIIs may be allowed to hedge their currency risks through exchange traded currency futures.
KYC norms are being simplified for Foreign Portfolio Investors.
To encourage longer-term flows and reduce volatility, FPI investments in G-Secs will henceforth be permitted only in dated
securities of maturity one year and above, and existing investments in T-bills will be allowed to taper off on maturity/sale.
Any investment limits vacated at the shorter end will however be available at longer maturities, so overall FPI limits will not
be diminished.
Q. Should we have differentiated licenses?
Pros
This will allow people to develop banking capabilities even with relatively small size of operations, which will then allow
them to may be apply for full banking licenses down the line.
Q. Does it make sense to augment FX reserves at this stage?
If you focus only on reserves there is really no point at which you feel safe, because provided there is enough uncertainty
about the economy, uncertainty about conditions, uncertainty about the treatment of international investors, 400, 500, 600
any level of reserves, until you get to Chinese levels, is probably not enough.
So really our focus should be on creating the policy environment which gives investors confidence.
Our intervention in exchange markets have historically been to reduce exchange rate volatility. So to the extent that we
have to intervene to prevent that kind of volatility, we have plenty of reserves.
6/23/2014 Evernote shared notebook:
https://www.evernote.com/pub/crazyphoton/interviewprep#st=p&n=7a5f4d76-f67a-42ea-8484-e7aaa3d53c47 6/34
have to intervene to prevent that kind of volatility, we have plenty of reserves.
Five Pillars of RBI's Developmental Measures
Clarifying and strengthening the monetary policy framework.
Strengthening banking structure through
new entry, branch expansion, encouraging new varieties of banks,
and moving foreign banks into better regulated organisational forms.
Broadening and deepening financial markets and increasing their liquidity and resilience.
Expanding access to finance to small and medium enterprises, the unorganised sector, the poor, and remote and
underserved areas of the country through technology, new business practices, and new organisational structures; that is,
we need financial inclusion.
Improving the systems ability to deal with corporate distress and financial institution distress by strengthening real and
financial restructuring as well as debt recovery.
Global Economy
Since January 2014 statement, global growth outlook remains broadly unchanged though weaker initial data to some
extent cloud optimism.
Global economic activity had strengthened in H2 of 2013. On the current reckoning, global growth is likely to be in the
vicinity of 3 per cent in 2014, about a percentage point higher than in 2013.
Downside risks
Tapering of quantitative easing (QE) in the US,
Continuing deflation concerns and weak balance sheets in the euro area and,
Inflationary pressures in the emerging market and developing economies (EMDEs).
Weakening growth and financial fragilities in China.
Capital flows to EMDEs could remain volatile, even if they do not retrench.
Indian Economy
Economic Buffers
These buffers effectively bulwarked the Indian economy against the two recent occasions of spillovers to EMDEs the
first, when the US Fed started the withdrawal of its large scale asset purchase programme and the second, which followed
escalation of the Ukraine crisis. On both these occasions, Indian markets were less volatile than most of its emerging
market peers.
With the narrowing of the twin deficits both current account and fiscal as well as the replenishment of foreign exchange
reserves, adjustment of the rupee exchange rate, and more importantly, setting in motion disinflationary impulses, the
risks of near-term macro instability have diminished.
Agriculture sector witnessed record production.
Industrial growth stagnating
IIP showed -0.1% during April-Mar 2013-14. Mining -0.8%, Manufacturing -0.8%. Capital goods: -4%, Consumer durables:
-12.2%. Growth of core industries remained sluggish at 2.6% during April-Feb 2013-14 compared to a growth of 6.4% in
the corresponding period a year ago.
Reduction in excise duty is expected to provide some relief to manufacturing.
Employment scenario showing signs of gradual recovery.
External trade
Apr - March: Exports: $312 bio (+3.9%), Imports: $450 bio (-8.1%), Trade deficit: -$138 bio (vs -$190 bio last year). Oil
imports: $167 bio (+2.2%), non oil imp: $283 bio (-13.3%). Agriculture exports: $45 bio.
March 2014: Exports: $29.5 bio (-3.2%), Imports: $40 bio (-2.1%), Trade deficit: -$10,5 bio (5 month high).
April 2014: Exports: $26 bio (+5%), Imports: $36 bio (-15%). Trade deficit: $10 bio. Oil imports: $13 bio (-0.6%), Non oil:
$23 bio (-21.5%).
Slowdown in exports in recent months can be attributed to certain sector specific issues and global factors. For instance,
decline in exports of gems and jewellery could be largely reflective of the price effect mainly emanating from an 20.1 per
cent y-o-y drop in gold prices.
Fall in exports of petroleum products are largely attributed to lower gross refining margins
Destination-wise, while export demand from economies like the US and China was broadly intact, a significant decline
was evident in exports to EU economies, Switzerland, the OPEC region, Singapore and Hong Kong SAR.
Although the decline in imports bodes well from the perspective of a CAD decline, the lowering of CAD on this account
may not sustain with the expected revival of domestic aggregate demand.
Surge in capital inflows led to accretion of reserves; the rupee has moved in a narrow range.
Inflation
CPI: 8.6% in April, 8.31% in March, 8.1% in Feb, 11.2% in Dec.
Decline mainly due to declining vegetable prices. Apart from vegetables, CPI inflation in cereals and products posted a
significant decline at 9.9 per cent in February 2014 from 12 per cent in November 2013.
Wage price spiral pushed up inflation in the services segment.
Headline inflation has moderated in recent months, but upside risks remain in 2014-15.
WPI: 5.2% in April (lower due to vegetables), 5.7% in March (higher due to food items), 4.68% in Feb, 6.16% in Dec.
Core inflation: 3% (highest since Apr 13).
Efforts to address infrastructure bottlenecks have yielded modest revival so far
Cabinet Committee on Investment (CCI) and the Project Monitoring Group (PMG) had together undertaken resolution of
6/23/2014 Evernote shared notebook:
https://www.evernote.com/pub/crazyphoton/interviewprep#st=p&n=7a5f4d76-f67a-42ea-8484-e7aaa3d53c47 7/34
Cabinet Committee on Investment (CCI) and the Project Monitoring Group (PMG) had together undertaken resolution of
impediments for 296 projects with an estimated project cost of `6.6 trillion.
However, 15-20 per cent of these projects, mostly in roads, power and petroleum, have reported additional delays, for
which the dates of completion have been extended further.
Also, there has been an increase in the number of projects without date of commissioning, mostly in roads reflecting the
growing uncertainty about their completion.
While fiscal targets were met in 2013-14 (RE), the quality of fiscal adjustment needs improvement.
IV. Differentiated Bank Licensing- Examining Pros and Cons
A. Arguments in Favour of Adopting a Differentiated bank Licensing
4.1 With the broadening and deepening of financial sector, it is observed that banks are slowly migrating from a situation
in the past where the number of banking services offered by the banks was limited and all banks provided all the services
to a situation where banks are finding their niche areas and mainly providing services in their chosen areas. Many banks
keep the plain vanilla banking as a small necessary adjunct. It is widely recognized that banks providing services to retail
customers have different skill sets and risk profiles as compared to banks which mainly deal with large corporate clients.
The present situation where every bank can carry out every activity permissible under Section 6 of Banking Regulation
Act, 1949 has the following implications, relevant to the subject under consideration :
For a wholesale bank dealing with corporate clients only, it becomes a costly adjunct to maintain a skeleton retail banking
presence. Moreover it becomes difficult for such a bank to meet priority sector obligations and obligations for doing
inclusive banking.
Retail banks may have to create risk management and regulatory compliance structures which are more appropriate to
wholesale banks, thus resulting in non-optimal use of resources.
Similar supervisory resources are devoted to banks with different business profiles. This may also result in non-optimal
use of supervisory resources.
The priority sector lending regime for foreign banks indicated in paragarph 3.3 has been causing some discomfort for some
of the foreign banks. For example, some of the foreign banks find it difficult to fulfil even the less rigorous target of 32 per
cent in respect of priority sector advances.
Some banks find it difficult to provide ' no frills' facility to economically disadvantaged. For them the more liberal licensing
regime causes a different set of problems.
It appears that given an opportunity, some of the banks may like to follow a niche strategy rather than competing as full
service all purpose banks.
2. On the other hand, there are some factors which point towards desirability of continuing with the existing system of
universal banking:
In India, the penetration of banking services is very low. Less than 59 % of adult population has access to a bank account
and less than 14 % of adult population has a loan account with a with a bank. Under such circumstances, it would be
incorrect to create a regime where banks are allowed to choose a path away from carrying banking to masses.
Priority sector lending is important for banks. The revised guidelines on priority sector lending have rationalized the
components of priority sector. For the first time, investments by banks in securitised assets, representing loans to various
categories of priority sector, shall be eligible for classification under respective categories of priority sector (direct or
indirect) depending on the underlying assets, provided the securitised assets are originated by banks and financial
institutions and fulfil the Reserve Bank of India guidelines on securitisation. This would mean that the banks' investments
in the above categories of securitised assets shall be eligible for classification under the respective categories of priority
sector only if the securitised advances were eligible to be classified as priority sector advances before their securitisation.
These measures would make it easier to comply with the priority sector lending requirements by those banks which had
faced some difficulties in this regard.
The business model adopted by such niche banks depends heavily on ample inter-bank liquidity. Any shock leading to
liquidity crunch can translate into a run on the bank. This situation has been clearly illustrated recently in UK in the case
of Northern Rock Bank.
Q how to draw private investments to manage problem to storage of perishable commodities?
One way is FDI in multi brand retail. But for this we need:
Policy certainty.
Reduce red tape.
Not to frame rules which discourage companies.
6/23/2014 Evernote shared notebook:
https://www.evernote.com/pub/crazyphoton/interviewprep#st=p&n=7a5f4d76-f67a-42ea-8484-e7aaa3d53c47 8/34
Not to frame rules which discourage companies.
Another is to boost local entrepreneurship.
Create a favorable ecosystem - ESMA, credit, reduce red tape, give more incentives.
National Food Processing Mission - Mega Food Parks.
Q. GDP of your town
$25 bio.
Q. UPA - 2 is failure ? comment
Result has been good in certain areas and not good in certain other areas.
Success
Social indicators: Poverty, IMR, education.
NREGA - empower rural workers.
RTI.
Failures
GDP growth, stalled investments.
Corruption, lack of transparency.
GST and other critical bills.
Foreign policy.
Welfare Indicators
HDI
A long and healthy life: Life expectancy at birth
Education index: Mean years of schooling of adults and Expected years of schooling of 5 year old children
A decent standard of living: GNI per capita (PPP US$)
Indian rank is 136 out of 187 countries @ 0.554 (slight improvement of 0.007 from last year).
Inequality adjusted HDI is 0.392 and rank is 91 (improvement from 93 last year).
HDI 0.800 indicates high human development.
0.799 HDI 0.500 indicates medium human development.
HDI < 0.500 indicates low human development.
CPI (corruption perception index )
India's rank s 94 out of 177 and is same as last year. Index is maintained by Transparency International.
The CPI scores and ranks countries/territories based on how corrupt a countrys public sector is perceived to be. It is a
composite index, a combination of surveys and assessments of corruption, collected by a variety of reputable institutions.
0 means that a country is perceived as highly corrupt and a 100 means that a country is perceived as very clean.
Ginni Coefficient
It is computed from the National Sample Survey for 2011-12. In rural areas, the coefficient rose to 0.28 in 2011-12 from
0.26 in 2004-05 and to an all-time high of 0.37 from 0.35 in urban areas, the figures showed.
India's Gini is 34 and ranks in moderately unequal countries.
MDG
Eighteen (18) targets were set as quantitative benchmarks for attaining the 8 MDGs.
Out of the 18 targets, 12 targets are relevant to India.
Target No. Target Description Progress
1. Halve, between 1990 and 2015, proportion of population below national
poverty line
On track
2. Halve, between 1990 and 2015, proportion of people who suffer from
hunger. Measured by underweight children under 3.
Slow or almost
off track
3. Ensure that by 2015 children everywhere, boys and girls alike, will be able
to complete a full course of primary education
On track
4. Eliminate gender disparity in primary and secondary education, preferably
by 2005, and in all levels of education no later than 2015
On track for
primary and
secondary, and
off track for
higher education
5. Reduce by two-thirds, between 1990 and 2015, the under-five mortality
rate
Moderately on
track
6. Reduce by three quarters, between 1990 and 2015, the maternal mortality
ratio
Slow or off track
7. Have halted by 2015 and begun to reverse the spread of HIV/AIDS On track
8. Have halted by 2015 and begun to reverse the incidence of malaria and
other major diseases
Moderately on
track
9. Integrate the principles of sustainable development into country policies
and programmes and reverse the loss of environmental resources. Energy
density of GDP, CO2 emissions per capita, forest cover.
Moderately on
track
6/23/2014 Evernote shared notebook:
https://www.evernote.com/pub/crazyphoton/interviewprep#st=p&n=7a5f4d76-f67a-42ea-8484-e7aaa3d53c47 9/34
density of GDP, CO2 emissions per capita, forest cover.
10. Halve, by 2015, the proportion of people without sustainable access to
safe drinking water and basic sanitation
On track for
drinking water,
slow for
sanitation
11. By 2020, to have achieved, a significant improvement in the lives of at
least 100 million slum dwellers
Not statistically
determinate
12. In cooperation with the private sector, make available the benefits of new
technologies, especially information and communication. Internet,
mobiles, computer penetration per 100 people
On track
Malnutrition among children (goal 2) is expected to reduce to only 33% as against a target of 26% by 2015.
IMR target is 27 per 1000 live births and India is expected to miss it.
MMR target is 109 per 1 lakh live births and India is likely to achieve only 139.
Target for drinking water was 17% and has already been achieved. India is likely to achieve universal safe drinking water
by 2015.
Sanitation target was to reduce non sanitation households to 38%. India currently has 49% non sanitation households and
is likely to achieve only 43% by 2015.
UNDP report on India's poverty
The estimated number of $1.25 poor in India in 2010 falls from 396 million in 2004-05 to 148 million in 2014. In 2014, the
World Bank reported that 11.8%[1]of all people in India fall below the international poverty line of US$ 1.25 per day (PPP).
Q. Interim budget vs Vote on account
Vote-on-account deals only with the expenditure side of the government's budget. The government gives an estimate of
funds it requires to meet the expenditure that it incurs during the first three to four months of an election financial year until
a new government is in place.
A vote-on-account is different from an interim Budget as the former is a statement of only expendiatures while the latter is
a complete set of accounts, including both expenditure and receipts. However, a vote-on-account is practised every
budget. Unlike a full Budget, the vote-on-account does not tweak the prevailing tax rates. Also, it cannot announce any
new schemes.
Eminent Indian Economists
K N Raj
He played an important role in India's planned development, drafting sections of India's first Five Year Plan, specifically the
introductory chapter when he was only 26 years old. He was a veteran economist in the Planning Commission.
He helped shape the contours of what later came to be called the "Kerala model" of development - the co-existence of low
per capita income and very high physical quality of life indicators.
He worked out a plan to raise India's rate of savings in the post-Second World War period when the country was in need of
foreign aid.
He computed India's Balance of Payments for the first time for the Reserve Bank of India.
Raj was an advisor to several prime ministers from Jawaharlal Nehru to P.V. Narasimha Rao.
Q. How have the social indicatiors fared in last decade?
Poverty: from 37.2% in 2004-05 to 21.9% in 2011-12. Rural poverty decline 16% is more than urban poverty decline 12%.
Odisha (25%), Bihar (24%), Rajasthan (20%), MP (17%) have shown biggest poverty declines.
Employment
IMR
MMR
Education
Child malnutrition
Sanitation
Drinking water
Q. Which ministry announces MSP?
Ministry of Agriculture
Q. You have heard of the poverty line, have you heard of something called the empowerment line?
Empowerment Line by Mckinsey is an analytical framework that determines the level of consumption required to fulfill
eight basic needsfood, energy, housing, drinking water, sanitation, health care, education, and social securityat a
level sufficient to achieve a decent standard of living rather than bare subsistence.
India's empowerment line at Rs 1,336 per capita per month. In 2012, some 56% or 68 cr Indians lived below empowerment
6/23/2014 Evernote shared notebook:
https://www.evernote.com/pub/crazyphoton/interviewprep#st=p&n=7a5f4d76-f67a-42ea-8484-e7aaa3d53c47 10/34
India's empowerment line at Rs 1,336 per capita per month. In 2012, some 56% or 68 cr Indians lived below empowerment
line - 2.5x the poverty line.
The Empowerment Gap, or the additional consumption required to bring these 680 million people to the level of the
Empowerment Line, is seven times higher than the cost of eliminating poverty as defined by the official poverty line.
If Indias recent weak economic performance continues and no major reforms are undertaken, we project that in 2022 more
than one-third of the population will remain below the Empowerment Line and that 12 percent will remain trapped in
extreme poverty.
To lift people above empowerment line, we need
Job creation and productivity gains have historically been the most powerful forces for improving living standardsand
India is in need of deep reforms that can encourage businesses to invest, scale up, and hire.
Raising farm productivity.
Increasing public spending on basic services. Additional resources will be available if India grows fast.
Make this spending and basic service delivery more effective.
This could lift 580 million people above the empowerment line, leaving 100 million (7 per cent of the population) below it in
2022 and 17 million (1 per cent) below the official poverty line virtually eliminating extreme poverty in just a decade.
Q. Agriculture-- in how much area, amount of food grain production--break up in rice, recent trends in food grain prod.
A crop year is from July to June. Total food grain production in 2013-14: 263 MT. 2012-13: 255 MT, 2011-12: 259 MT.
Good monsoon in 2013 helped farmers increase area under cultivation in both kharif (summer) and rabi (winter) seasons in
the current crop year.
The table below gives estimates of major crops in million
tonnes.
-------------------------------------------------------
CROP 2013/14 2012/13
-------------------------------------------------------
Rice 106.19 105.24
Wheat 95.60 93.51
Corn 23.29 21.06
Pulses 19.77 18.34
Tur 3.34 3.02
Gram 9.79 8.83
Total food grains 263.20 257.13
Total Oilseeds 32.98 30.94
Sugarcane 345.92 341.20
Cotton #35.60 34.22
------------------------------------------------------
Q. What is the growth rate of agriculture? What are the export / import trends?
Indian exports were $24 bio whereas imports were $11 bio giving a surplus of $13 bio. Cotton, marine products, oil meals,
basmati rice and sugar are main exports. Main imports are vegetable oils, wood and pulses.
Agriculture growth: 2.5% in 2011-12, 7.0% in 2010-11. In 1980s it was 3%, in 8th Plan it was 4.8%, in 9th and 10th Plans
it was 2.5%. 11th Plan had a growth of 3.2%.
Q. Laws made by govt are hindrance to economic growth ??
Laws are necessary - Rule of law, contract enforcement is absolutely necessary for economic development.
Excessive red tape, discretionary decision making, retrospective amendments are hindrance to economic growth.
Q. Which renewable energy source holds most promise for india?
Wind
The global practice is now to build towers in the range of 80120 m, which significantly increases the power potential. At
the same time, the size of wind turbines has increased while the earlier turbines were typically less than 1 MW, the
recent designs go up to over 5 MW.
Taking these into consideration, the wind potential in India is now estimated at about 103000 MW for 80 m hub height.
Wind potential is unevenly distributed across the country; only Karnataka, Tamil Nadu, Andhra Pradesh, Maharashtra and
Gujarat have significant potential.
Wind power has significant seasonal and even intra-day variations. Wind capacity addition needs to be complemented by
other energy sources, which have a quick ramp-up time.
M3: What is insider trading? What was the case of Rajat Gupta and Goldman Sachs?
M3: What are the safeguards in India regarding insider trading?
Q. GST and DTC - what are the problems?
6/23/2014 Evernote shared notebook:
https://www.evernote.com/pub/crazyphoton/interviewprep#st=p&n=7a5f4d76-f67a-42ea-8484-e7aaa3d53c47 11/34
Q. GST and DTC - what are the problems?
GST
Compensation of states.
Flexibility on rates.
Dispute resolution mechanism.
************************************
FSLRC Recommendations
Nachiket Mor Committee
It was appointed by RBI for financial inclusion for Small Businesses and Low-Income Households.
(a) Every adult (Above 18 years) of our country should have a bank account by January 1, 2016. This account will be
known as Universal Electronic Bank Account (UEBA).

(b) Every resident should be issued an account at the time of receiving Aadhaar number (UIDAI) by a bank itself.

(c) It recommends unified Financial Redress Agency under Finance Ministry for customer grievances.

(d) It recommends abolition of interest subsidies and loan waivers. It suggested that government should transfer benefits
directly to farmers.

(e) Permission to banks for pricing farm loans below base rate should be withdrawn.

(f) Statutory liquidity ratio has outlived its utility for both Banks and NBFCs. So, it needs to be scrapped.

(g) It recommends raising priority sector lending cap for banks to 50 per cent from the current 40 per cent.

(h) It also proposed for creation of a Payment Bank (PB) to provide payments services including credit, insurance and risk
management products.

(J) Panel also suggested for the creation of State finance regulatory commission (SFRC). All existing regulators at state
level should be merged into SFRC.

(k) Each districts should have a total term life insurance sum assured to GDP ratio of at least 30%.
Two New Institutions
The Mor Committee favours the creation of a payments bank.
Payments bank
The rationale is that a large number of households may not need loans but would like to access the formal banking
system purely for the purpose of making payments or withdrawals.
Proof of this need, according to the committee, lies in the fact that prepaid issuers (PPIs), such as Airtel Money have
found a large market for their services.
The committee believes there are two problems with the PPI model. One, it exposes the sponsor bank to the risks of a
run on the PPIs, while the PPIs have to take a view on the quality of the sponsor bank. Two, since PPIs do not pay
interest on the cash balances, it inhibits use of such payment services. A payments bank would address both these
problems.
Priority sector lending
Sector wise weighting
The rationale for a sector-wise weighting is that, in the different segments of PSL (agriculture, small and medium
enterprises (SMEs), exports and weaker sections), banks meet their targets for some components but not for others.
6/23/2014 Evernote shared notebook:
https://www.evernote.com/pub/crazyphoton/interviewprep#st=p&n=7a5f4d76-f67a-42ea-8484-e7aaa3d53c47 12/34
enterprises (SMEs), exports and weaker sections), banks meet their targets for some components but not for others.
Wherever there are shortfalls in a given segment, lending to that segment should have a weight of more than one so that
lending is incentivised.
The committee argues that not all banks need to have the same mix of lending towards PSL. Banks can focus on
segments consistent with their expertise and meet an adjusted PSL target of 50% in ways that suit them. Urban banks
can focus on SMEs; rural banks can focus on agriculture.
Saumitra Chaudhri Committee on IIP
It has called for complete overhaul of the index of industrial production (IIP),
The panel has recommended that the constituent items in the index, and their weights, be revised annually rather than
after five years.
To address the issue of large swings and volatility in such heads as producer goods, the committee has proposed
estimating capital goods output on the basis of expenditure incurred towards production each month, rather than focus on
physical aggregates. This also seems to make sense, as, for lumpy capital goods items like turbines and boilers, it can
take several months to complete production.
Further, put out seasonally-adjusted industrial data on a month-on-month basis, which is as per global practice.
Also bring out an index of services output.
*******************************
Bihar vs Gujarat or Kerala vs Gujarat
Pro Gujarat
- The accompanying chart shows that Gujarats economic growth in 2001-2011 at an annual average rate of 10.2% was
2.5 percentage points higher than the national average. A closer look at the chart shows that Gujarats growth in the prior
decadebefore Modi took charge in 2001at 7.5% also beat the national average, albeit by a slightly smaller margin of
1.4%.
- Gujarats agricultural performance was superior to most other states even in the 1990s. There is a remarkable
turnaround of the resource-poor and drier regions within Gujarat (such as Saurashtra) during this period. This is due to
watershed management, improved irrigation services and rural infrastructure including electricity supply. The
commissioning of the Sardar Sarovar Project (SSP) also aided farm growth by raising the irrigation cover.
- Between 2004-05 and 2011-12, overall poverty in Gujarat fell by 15.2 percentage points compared with 15.3 percentage
points nationally. Given that the poverty ratio in Gujarat in 2004-05 was already 5.4 percentage points below the national
average, this progress is nothing to scoff at.
- Gujarat also deserves applause for the large cuts in poverty among the Scheduled Castes (SCs) and Muslims. At 7.7
percent, Gujarat now boasts the lowest poverty ratio for Muslims in rural areas.
Pro Bihar
- In stark contrast, Bihars economy limped along at an average annual growth rate of 2.7%, 3.4 percentage points slower
than the national average, in 1991-2001. Over the next decade though, Bihars growth rate trebled to 8.2%, beating the
national average for the first time in independent Indias history, even though the improvement over the national average
was by a small margin. Between 2005when Kumar took charge of Bihars governmentand 2011, the state grew at an
annual pace of 10.9%, beating both the national average and Gujarats growth rate of 9.6% during this period.
- While Gujarats HDI performance was above the national average in the 1980s and 1990s, it decelerated in the 2000s
and came down to the national average. In contrast, Tamil Nadu and Maharashtra, which started off at a similar level of
HDI as Gujarat in the 1980s, have continued to perform better than the national average in the 2000s. Bihar, on the other
hand, has consistently been below the national average, but has made significant improvements over the last decade and
shows signs of catching up to the national average.
Balanced View
- There are areas where Bihar government needs to chart different course of action borne out of Bihars complex socio-
economic realities. While schemes like Bicycle Yojana, Poshak Yojana ,Dress yojna,Award for passing Matriculation
Examination might not be required for states like Kerala, TN or even Gujarat, still it would make imminent sense for Bihar
as State is on bottom of literacy rate.
- Need to draw the good points from other states as well. Eg. Right to Public Services from Bihar, MP. Fast Track Courts-
from Bihar.
6/23/2014 Evernote shared notebook:
https://www.evernote.com/pub/crazyphoton/interviewprep#st=p&n=7a5f4d76-f67a-42ea-8484-e7aaa3d53c47 13/34
from Bihar.
Government policies
That said, the challenges posed, directly or indirectly, by government policies are formidable. Consider the following.
Difficult business environment: India ranked 132 out of the 185 countries in the World Banks Doing Business survey in
2013. According to the official data, nearly 70 clearances are required annually for businesses to operate. The greatest
cost falls on SMEs, where the proprietor has to bear the entire burden. Such an environment, combined with retroactive
changes in tax demands, creates much uncertainty, anathema for investment. Another challenge is the lack of adequate
protection against extortion and protection rackets.
Labour deployment rigidity: Indian manufacturing has suffered in the past from the twin constraints of militant and
competitive trade unionism and a plethora of labour legislation. In recent years, unionism has ostensibly weakened.
Nevertheless, it is still present in major industrial centres, and its infrequent but violent demonstration discourages foreign
investors and induces others to keep employment to a minimum. India has nearly 50 laws at the central or state level that
affect labour conditions. Consequently, hardly any enterprise can claim to be in total compliance.
Infrastructure deficit: The peak power deficit in India is estimated at 7% to 8%, and industry is not insulated from the
resultant power cuts that sweep the country. Most large manufacturing units have had to create full backup capacity,
raising capital costs. Indian companies across the board bear a significantly higher price for infrastructure services and
utilities than their global competitors.
Regulatory delays and lack of transparency: Over time, a rather complex regulatory structure has been established to deal
with land acquisition, land use and the environment. The process has become increasingly time-consuming, opaque and
unpredictable, especially during the past 10 years under the rule of the United Progressive Alliance. According to one
survey, 1,240 central and state regulations apply to the industrial sector.
Iron ore mining Goa
The Supreme Court allowed an annual cap of 20 million tonnes of iron ore to be extracted in Goa which was banned by it
in the state for nearly one-and-a-half years.
It also said there will be no grant of lease for mining around one km of national parks and wild life sanctuaries. The court
directed the Ministry of Environment and Forests (MoEF) to identify eco-sensitive areas around national parks within six
months.
It said the Goa government will formulate a scheme within six months for utilising the funds generated by e-auction.
Workers on rolls of all mining firms will be paid 50 per cent of the wage during the period for which they were out of work
because of the apex court stay on the mining activity.
Q. What are the provisions of the Dodd-Frank Bill / Volcker Rule?
Proprietary trading is completely banned.
Banks investment in hedge funds and private equity should be the lesser of 3% of total fund assets and 3% of its Tier I
equity capital and even these exposures are subject to a number of safeguards.
Banks cannot deal in swaps below investment grade.
US banks are expected to comply by July 2014.
Rules on executive pay - clawback, rejection of golden parachute.
6/23/2014 Evernote shared notebook:
https://www.evernote.com/pub/crazyphoton/interviewprep#st=p&n=7a5f4d76-f67a-42ea-8484-e7aaa3d53c47 14/34
financial stability council - at the top of other regulatory bodies to reduce regulatory gaps.
many entities were unregulated. they will be brought under net.
consumer protection.
Q. What are the recommendations of Vickers Commission?
Creates ring fenced entities.
mandated banking activities: taking deposits from and making loans to individuals and SMEs.
prohibited activities. These would include trading, purchase of loans and securities, transactions outside the European
Economic Area and with a non-ring-fenced bank.
permitted activities. taking deposits from customers other than individuals, SMEs and lending to large companies.
This is the location of the fence.
Then, there is the height of the fence. The ring-fenced entity would be a separate legal entity with its own board of
directors and making disclosures as though it were an independent listed entity. The relationship of the ring-fenced retail
banking entity with other entities in a wider corporate group should be conducted on a third-party basis and it should be
able to meet its requirements of capital and liquidity on its own.
Finally, the ring-fenced entity would have higher capital requirements than required under Basel 3. The commission
recommends equity capital of 10% for ring-fenced banks with risk-weighted assets of more than 3% of the UK GDP; total
capital would be in the range of 17-20%.
Q. What are Basel III requirements?
Economics
1. direct cash transfer... benefits vs limitations
Benefits
obvious - cut intermediary chain. less corruption, public money well spent.
reduce economic inefficiency. targeted subsidies create distortion between product choices. This gives freedom to hh to
spend it as it likes.
overhaul systems - like electricity - no need to give coal mines for free. can develop a highly competitive supply chain. this
would also increase renewable energy.
Limitations
women and child rights may suffer.
network density of banks, post offices less than FPS. literacy an issue in dealing there. so financial inclusion is a pre
condition.
volatility in prices of food - but cash subsidy will be slow to change.
inflation is NOT a limitation.
2. food security bill implementations issues
Implementation issues in food security bill
identifying 67% population (75% rural, 50% urban). socio economic caste census not completed so far. 5 kg @ Rs. 2 for
wheat and Rs. 3 for rice. identification to be left to states.
large investment needed to create infra first.
contradiction with directs benefits transfer.
states with universal or quasi universal PDS suffer. Poor states also suffer in theory.
last mile management left to local authority and not local government.
govt has agreed to maintain existing state food grain quotas and also antodaya (35 kg pm @ Rs. 2 for wheat and Rs. 3 for
rice) - not a limitation.
3. super tax on rich - justified? on related note... justification for a capital gains tax and plugging mauritius tax treaty?
Super tax
fundamentals of taxation - ability to pay - justified.
argument given - reduce incentives to invest. flawed - most of the income is in form of capital gains which won't be taxed
anyways. this will only tax the salaries they draw.
argument given - reduce incentives to work harder - why is it any different from any general tax increase..
In india only a small proportion of rich declare their true income. so unfair on them. but proper solution is to get remaining
to declare true income.
Capital gains tax
fundamentals of taxation - vertical and horizontal equity. if we exempt capital gains, both equity sacrificed.
argument - reduce incentives to invest, capital flight, india is a capital scarce country. But US is a capital rich country,
why do they need to exempt capital. its not that all the capital will fly away. investment decision involves both profit and
6/23/2014 Evernote shared notebook:
https://www.evernote.com/pub/crazyphoton/interviewprep#st=p&n=7a5f4d76-f67a-42ea-8484-e7aaa3d53c47 15/34
why do they need to exempt capital. its not that all the capital will fly away. investment decision involves both profit and
cost considerations. profits will be reduced to the extent of capital gains tax. so only investment at the margin will not
happen. we can simultaneously bring structural reforms to increase the profitability of the economy.
argument - capital at margin goes away - many considerations - for example elasticity of this capital at margin... have to
see what that capital was doing in the first place, how many jobs it was generating - the tax proceeds can be used to
create more jobs than lost... considerations of inclusive growth.
why should speculation in stock markets be exempted... what significant gains does it bring?
argument - double taxation.
argument - horizontal inequity.
Mauritius treaty
additional argument given - finance CAD... but for short term flows we need to keep fx reserves... those fx reserves can't
be used to finance imports. so no use.
distortion with domestic investors.
4. gas pricing formula of rangarajan - pros and cons..
Pros
market efficiency - also encourage new production.
low monitoring and litigation costs for government.
Cons
Higher subsidy burden as government subsidises upstream plants. this can be recouped partially if producers are taxed
more.
5. coal price pooling - good or bad...
it will lower the marginal cost for new plants and thus will encourage new investments. earlier marginal rate was import
price, now the pooled price.
it will save investments already made. but will increase costs to those who were already getting cheap coal from CIL.
Not long term solution
as imported coal goes up, pooled price will increase. so need to increase domestic production.
Should we free up domestic coal prices?
yes - efficiency reasons - more production of coal.
But will this not increase the cost of electricity? power producers must be allowed to pass on the increased cost. the
discoms should buy power at a higher cost and if the government wishes to subsidise a section, it must pay discoms
directly or use direct benefits transfer. the additional money can be obtained through auction of coal mines. this system is
more efficient and transparent at all levels.
yes - if we heavily subsidise coal power, how will we ever develop renewable resources?
no - players with existing captive mines will have windfall gains.
6. is 5.5 the new secular hindu rate of growth?
post crisis GFCF has fallen 4-5% from pre crisis level. pre crisis growth was 15-16%, now only 5-6%. this clearly shows
that producers don't have confidence in the recovery and the growth in 2 years after the crisis was merely a restocking
growth.
no - do we think that for next 10 years external environment will remain so subdued? external environment has a big role to
play in this fall. there was global recovery for 2 years and then 2012 was again a difficult year globally. even china recorded
just 8% growth this year.
structural issues had always been there and yet growth was high pre crisis.
no - reforms are underway. land acquisition, subsidy reforms, governance reforms, human capital development.
no - fiscal consolidation is underway. IMF research shows crowding out effects more pronounced in india.
Importance of exports
The consumption is projected to decrease as a proportion of GDP and savings to increase. This savings has to serve
exports. So marginal growth export led.
7. reasons behind falling growth rate.
shaken investor confidence - post crisis GFCF has fallen 4-5% from pre crisis level. pre crisis growth was 15-16%, now
only 5-6%. this clearly shows that producers don't have confidence in the recovery and the growth in 2 years after the
crisis was merely a restocking growth.
global factors - importance of global factors because of importance of exports.
importance of structural reasons - but they have been there all the time and yet pre crisis growth was high. But obviously
structural reforms will be able to pull up growth level.
higher fiscal deficit - crowding out.
8. reasons behind high inflation
currency depreciation.
higher fiscal deficit.
higher incomes - so naturally any index which has higher weightage of food will show higher inflation.
So is NREGA bad as it causes food inflation?
If we take only a very narrow outlook, then can't deny. But need a proper perspective to evaluate it.
food inflation - what is the use of keeping food prices low if you let 40% of the population starve? N Korea example. what
gives us this special right? don't they have a right to eat?
talk about productivity enhancement potential - small and marginal farmers.
6/23/2014 Evernote shared notebook:
https://www.evernote.com/pub/crazyphoton/interviewprep#st=p&n=7a5f4d76-f67a-42ea-8484-e7aaa3d53c47 16/34
talk about productivity enhancement potential - small and marginal farmers.
9. reasons behind high CAD
slowdown in exports. happening everywhere - decoupling doesn't hold.
oil - prices in $ have gone up + car argument.
gold - marginal investment demand - bull market in gold + domestic inflation + bad time in financial markets.
10. steps taken to curtail gold import and are these steps really going to curtail the imports?
duty hikes + RBI working group proposals (gold banks + gold deposit schemes + bullion corporation of india - refinance
gold loans and pooling) + spreading financial literacy and products.
duty hike have the potential of increasing smuggling. utilising domestic idle gold domestically is a good idea. in the long
term we need to check inflation and spread financial products.
11. why has rupee depreciation not improved CAD
high import content of our exports - petroleum products ($60 bio) + gems and jewelry ($45 bio).
sticky INR prices of imports - oil + fertilisers. inelastic demand.
investment demand in gold ($60 bio + $30 bio) - fueled as INR prices go up.
12. reasons for falling NREGA performance in rajasthan
13. Cabinet committee on investments - good or bad idea
overall needed - agency based system, outcome oriented. usually system is procedure based with a myopic view about
outcomes. and consensus is either not reached or is severely sub optimal. but care should be taken that environment and
tribal concerns are not bulldozed away.
better way is governance reforms - reduce discretionary powers, arbitrariness. strengthen outcome based system and not
procedure based - give performance based incentives.
14. taking away powers of gram sabhas in linear projects - good or bad idea
unfortunate, but necessary - land is not fungible. but rehab shouldn't be neglected - make rehab a mandatory requirement
before acquisition.
15. biggest advantages for india in coming decades.
human capital development. higher education - elite system to mass system. primary - universal enrollment.
demographic dividend + hidden demographic pool.
pressures build up for improved governance and incentive structures.
Biggest challenges to development
inequality - non inclusive growth - no human capital development.
crony capitalism - people will lose faith in governance, government will lose credibility, any reform becomes impossible.
16. should corporate houses be allowed to start banks?
Yes
large industrial houses are already involved in the NBFCs, why make a fuss about bank licences?
increased competition - PSB privatisation unlikely
strong moral hazard, 1969 rationale, but it can be checked through more effective regulation and monitoring. check in
house transactions, stricter CDR norms for inhouse restructuring. mandate a low ceiling on a companys loans
outstanding to the banking system to be eligible for a bank licence.
17. should stock exchanges be listed?
yes - more transparency, need capital to grow. but should not list on itself, holding should not be concentrated.
no - sharp fall in stock exchange's stock's price will adversely impact whole market. but if it is happening because of say a
fraud, then it would have anyways impacted the market trading when people come to know about the fraud + proper way is
better regulation. and if happening due to competition, economic reasons, then trading will simply shift to more efficient
exchanges.
18. aadhar card - is it a good idea.
yes - gives identity. through it we can implement direct benefits transfer which can be used to reform food, electricity,
fertilisers sector.
can track medical history.
useful for migrants.
wee out ghost beneficiaries.
Concerns
privacy. need to protect data from private players as well as unauthorized use by state.
parliamentary committee has raised financial issues, duplication issues, technology issues.
19. what are the main problems of industry these days
land acquisition and government clearances and control.
crony capitalism - brings uncertainties in government policies, benefits some - hurts others, promotes inefficiency.
poor infrastructure.
How can we resolve land acquisition issues
Biggest issue - people are not paid enuf. we expect to pay them nothing or a pittance and leave peacefully? guiding
principle should be that after rehab, people are not worse off + rehab has to precede acquisition.
won't that increase project costs, reduce investment and hurt economy?
where is investment taking place in the current system? projects are held up for 10 years... proper rehab will only
smoothen process, bring certainty and make investments easier.
if we pay proper rate for other resources, why not land? should an steel plant investor say he should get iron ore for free?
6/23/2014 Evernote shared notebook:
https://www.evernote.com/pub/crazyphoton/interviewprep#st=p&n=7a5f4d76-f67a-42ea-8484-e7aaa3d53c47 17/34
if we pay proper rate for other resources, why not land? should an steel plant investor say he should get iron ore for free?
human rights.
practical solution - rehabilitation cess on project beneficiaries.
20. what are the main problems of handicrafts industry in jaipur these days?
slowdown.
marketing issues - inefficient supply chain - leaves little with the artisan to invest.
dwindling skilled labor - as profession no longer remunerative, so people migrating to other sectors.
Steps taken
diversification of markets.
create efficient supply chains. more bargaining power to producers - SHG model.
cluster development for infra and technology.
21. How to build trust among people on rehabilitation and nuclear issues? (also find nuclear power world stats)
make people at least not worse off after rehab + rehab should precede acquisition.
levy rehab cess on project beneficiaries.
Nuclear issues
need more transparency. if govt scientist comes and says its safe, whats his credibility? need eminent and independent
experts.
22. NSDC architecture
23. What did 13th FC do to strengthen the PRIs.
It also called for a 2.5% share of the divisible pool for local bodies in the form of grants, as the Constitution does not allow
the sharing of tax revenues with them. While 1.5% will be constant, 1% will be on the basis of performance.
The commission has recommended that local self-government bodies be given a direct share in Union tax revenues,
bypassing the state governments, and that the cut for urban local bodies be raised in tune with their population growth vis-
-vis rural bodies. It is like bypassing the state, and the Centre will be directly dealing with the local self-governments
(LSGs). The LSGs will get their grants on the basis of their performance and their share from the tax revenues and states
will not have much powers on them.
The commissions recommendations also seek to balance the share of urban bodies in total revenue. While revenue is
currently divided between rural and urban bodies in the ratio 4:1, the commission has suggested raising the latters share
to match their share of the population according to the 2001 census.
Fiscal capacity distance criteria
We want to measure tax capacity. earlier income distance was used i.e. distance from state with highest per capita
income. But this assumes uniform tax / income rate. 13th FC changed this - said uniform rates not realistic. special
category states need to have different rates. (distance from harayana used - goa first).
24. lobbying - good or bad.
essential in democracy - freedom of speech. but rules should be there to make it transparent.
Lobbying in the United States describes paid activity in which special interests hire well-connected professional
advocates, often lawyers, to argue for specific legislation in decision-making bodies
25. Is auction always the best way to allocate resources?
a very good way though not optimum in all cases. there is no problem if any action is taken in a non corrupt way. we
should have rule based transparent systems.
26. Doesn't the exposure of these scams hurt industrial growth?
hurts - litigation, policy uncertainties, governance credibility. but alternative is worse.
proper way is to have transparent, rule based systems.
27. should there be a tax on diesel cars?
yes - misuse of subsidy, environmental concerns.
no - now is not the time because of slow growth.
28. Should STT be abolished? Why? Tobin tax?
Rationale
no - tax deduction at source. earlier people used to avoid capital gains tax.
yes - reduces overall market liquidity and some impact on employment. creates inefficiencies.
no - FIIs at least pay some tax.
Tobin tax
originally a transaction tax proposed on currency spot trades when bretton woods was dismantled. to discourage
speculation by short term trading.
29. are u suggesting welfare programs like NREGS to be scrapped to bring down inflation?
No. that would be taking a very narrow view. NREGS has increased incomes, naturally demand for food higher and any
index which has a higher weightage of food will show higher inflation. But shouldn't they eat? whats the point in keeping
6/23/2014 Evernote shared notebook:
https://www.evernote.com/pub/crazyphoton/interviewprep#st=p&n=7a5f4d76-f67a-42ea-8484-e7aaa3d53c47 18/34
index which has a higher weightage of food will show higher inflation. But shouldn't they eat? whats the point in keeping
prices low when 40% starve?
talk about productivity enhancement potential, small and marginal farmers, other benefits.
30. food production is increasing every year. But there is very high food inflation prevalent throughout the country why?
not rising enuf - demand going up more. rising incomes.
protein items like pulses, horticulture and fisheries - supply not going up much. yields are stagnated. supply chains
inefficient and farmers have little incentives and high risks.
31. Why are business ethics important?
business ethics act as a first level check in ensuring corrupt things don't happen. not sufficient - but important.
people are not taught business ethics. many times they don't realise what they are doing is wrong and thus contribute
towards the wrong. give example of convertible bonds.
32. foodgrain procurement pricing policy in india - different aspects
can be instrumental in solving many agro issues. for that it needs to be made effective for other crops.
guiding principle should be to shape cropping pattern as per the water and geographic conditions as well as changing
demand patterns.
at the same time basic level of incentives should be maintained.
33. NREGA different aspects.
generating employment - 1000 cr man days, 10 cr families.
generated a structural break in rural wages.
reduced push migration.
strengthened PRIs.
social justice - SC/ST, women.
gives support when most needed - adverse weather conditions.
productivity - s&m farmers, enterprises.
poor asset creation though - planning, lack of staff.
poor completion rates - reassess fund allocation for works when wages revised, reclassify works.
delayed wage payments - MIS, administrative delays in fund sanction, lack of staff.
34. Should CRR be done away with?
convenient tool for policy operation. the real issue is paying interest on it - to which i see no objection. simply will have to
conduct extra OMOs to sweep up the interest payment.
other tool could be simply using interest rates. but transmission mechanism has to be strong there. So OMO can be used
alongside.
35. Should RBI look at CPI or WPI while deciding monetary policy?
case for CPI - this is what anchors inflationary expectations since this is price actually paid. captures economy more
meaningfully than WPI. CPI gets reflected in wage settings.
case for WPI - core inflation argument. but then in a country where half the personal consumption is on food and fuel,
tough to ignore this component.
in current settings, a mix has to be used. track WPI but with certain limits for CPI.
36. should banks be privatised?
present structure of government ownership will soon become a big constraint for their growth.
but financial crisis tells us that government ownership is such a big cushion.
alternative ways could be explored - like issuing non voting shares + better regulation.
37. Is growth bottoming out?
Not in terms of data. IIP, WPI, trade. even global growth doesn't show turning signs.
but policy actions have reversed which is a brighter side.
38. can an undervalued currency help in long term?
no - as per economic theory. but give the limitation in the theory - assumption of no sterilisation.
Should one have very large fx reserves?
savings - sacrificing present consumption for a claim on future. good for certain basic things like macroeconomic stability.
but after a point, it may reduce welfare. if we cut down too much on our food today we may not be healthy enough to work
tomorrow. similarly if we don't burn that drop of oil today, our economy may not grow enough for tomorrow.
39. why has low growth failed to pull down inflation?
let us analyse why inflation is going up - 1. food inflation - divergence between CPI and WPI, rising incomes is a cause +
supply side reasons. 2. highlight manufacturing and core inflation already low.
global rally in energy and commodity prices - supply side shock.
weakening currency - highlight pro cyclical nature of INR - dependence on FII inflows. when growth low, our currency will
weaken and thus higher inflation.
infrastructure constraints lower production and increase prices.
40. why has low growth failed to pull down CAD
conventional theory.
But low growth a global phenomenon - so exports also hit.
then imports - highlight the case of oil and gold.
41. In low growth won't reducing fiscal deficit hurt even further?
crowding out argument - evidence of pre crisis.
6/23/2014 Evernote shared notebook:
https://www.evernote.com/pub/crazyphoton/interviewprep#st=p&n=7a5f4d76-f67a-42ea-8484-e7aaa3d53c47 19/34
crowding out argument - evidence of pre crisis.
need to ensure that cuts are not from capital expenditure. cuts are from revenue expenditure.
42. won't a cut in the plan expenditure hurt supply capacity of economy and reduce growth even further?
yes. so need cut in revenue expenditure or enhanced tax collections.
43. Should tax base be expanded?
top 1.5% pay 63% tax. cost vs collection tradeoff.
better results can come if we dedicate energies towards closing loopholes at top.
44. Is the economy turning around?
FM insists CSO estimates are wrong because he promised in HK that economy is turning around. So he says CSO fails
to see the green shoots in the economy. CSO advance estimates use linear extrapolation for last 4 months.
But IIP indicates green shoots are absent. December and November showed negative on a seasonally adjusted basis.
April - December growth is 0.7% only.
But in terms of policy actions, it is turning.
44. What are the new RBI CDR norms?
From April 1 onwards, fresh CDRs will be provisioned at five per cent. Provisioning will be increased in stages to five per
cent on the existing stock of CDRs, which are currently provisioned at 2.75 per cent.
Promoters must now commit to bring in the higher of two per cent of outstanding debt and 15 per cent of what the banks
are foregoing.
Further, lenders may only convert a maximum 10 per cent of debt into equity a clause that would have blocked deals
like Kingfisher Airlines, where lenders collectively converted over 20 per cent of debt into equity.
The RBI has also plugged loopholes in the reclassification of CDRs to standard, or non-NPA, describing this as the end
of regulatory forbearance. The existing guidelines allow a restructured NPA to be reclassified standard if the borrower
pays some part of outstanding interest. Under the new guidelines, the entire restructured interest due will have to be paid
for a year before reclassification.
The tightened definition implies that a large percentage of restructured loans will remain NPAs, which are provisioned at 15
per cent. Around 10 per cent of all PSB assets are in CDRs. Private sector banks are better off, with lower CDR exposure
at 1.5 per cent of assets.
Q8: What is a stock exchange? How does it work?
Current Macroeconomic Issues
Targets of 12th Plan
Taking account of all these factors, the Twelfth Plan should work towards bringing GDP growth back to an inclusive 9 per
cent in the last two years of the Plan, which will yield an average about 8.2 per cent in the Plan period. These are a growth
rate of 4 per cent for the agricultural sector over the Twelfth Plan period and around 10 per cent in the last two years of the
Plan.
Growth: 8.2%, mining and quarrying @ 7.2% up from 3% in 11 plan. manufacturing avg of 8% up from 6.9% in 11th
poverty: 2% p.a. mean schooling years from 5 to 7. eliminate gender and social gap in school enrollment.
increase gross irrigated area from 90 mha to 103 mha.
reduce AT&C losses to 20%
16. Connect all villages with all-weather roads by the
18. Complete Eastern and Western Dedicated Freight Corridors by the end of Twelfth Five
19. Increase rural tele-density to 70 per cent by the
50 per cent of gram panchayats achieve the Nirmal
2. Clean 80 per cent of critically polluted stretches in rivers by 2017 and 100 per cent by 2020.
green cover by 1 mha.
IMR 28, MMR 100
Skill development - 50 mm. special efforts are needed to ensure that employers and enterprises play an integral role
88 GW capacity, 30 GW renewable
reduce emission density by 20% 2020 vs 2005.
provide banking to 90% of households.
direct benefits transfer.
But it is reasonable to plan for merchandise exports growing at an average annual rate of 17 per cent in 12th Plan
compared to 20 of 11th Plan
6/23/2014 Evernote shared notebook:
https://www.evernote.com/pub/crazyphoton/interviewprep#st=p&n=7a5f4d76-f67a-42ea-8484-e7aaa3d53c47 20/34
compared to 20 of 11th Plan
CAD 2.9%. Investment in valuables has increased from 1-1.4% pre crisis to 2.5% now. This needs to be brought back to
1.5% level.
Plan size: 7% of GDP, Plan size of states: 5.5% of GDP
GFCF: 35% up from 32% Most of increase in GFCF has to come from private sector. This is possible if fiscal deficit is
kept under check to prevent crowding out.
To move from GFCF to gross capital forma- tion we need to add increase in inventory and invest- ment in valuables
Investment in Infra: 9%, 11th Plan raised it from 6% to 7%.
Savings and Investment Scenario
One was the big improvement in government finances and the other was the improvement in the level of retained earnings
of the private cor- porate sector. Between 200102 and 200708, the savings of government administration improved from
minus 6.0 per cent of GDP to plus 0.5 per cent of GDPan improvement of 6.5 percentage points. This was equal to
almost half of the 13.4 percent- age point improvement in the overall savings rate. The retained earnings of the private
corporate sec- tor improved from 3.49.4 per cent of GDPan increase of about 6.0 percentage points.
Gross financial savings by house- holds improved by 2.3 percentage points, but then so did the sectors liabilities
The decline in domestic savings rates after the crisis of 2008 reflects deterioration in precisely the two elements, which
had accounted for the increase earlier. this reduced the savings of the public sector by as much as 4.3 percent- age
points of GDP accounting for nearly two-thirds of the fall of 6.5 percentage points in the domestic.
private sector savings declined by 1.5%.
In the 12th plan bulk of the savings gain coming from public sector. from 0.7 to 4.3.
second largest is from domestic financial savings.
Household Savings
2.49. The gross financial savings of the household sector is expected to average 15.1 per cent in the Twelfth Plan going up
from 13.6 per cent in 201011 (and an estimated 12.4 per cent in 201112), to 16.0 per cent at the end of the Twelfth Plan
(201617). The borrowings of the household sector from the financial system are expected to increase from 3.6 per cent in
201011, and estimated 3.1 per cent in 2011 12 to 4.5 per cent in 201617. Thus, the net finan- cial savings of the
household sector is expected to go up from 10.0 per cent in 201011, and an estimated 9.3 per cent in 201112 to 11.5
per cent in 201617, while the average for the Plan period is likely to be 11 per cent. Investment by households in
physical assets is expected to average 12.3 per cent of GDP in the Twelfth Plan. Thus, the total household sav- ings
including both net financial and physical assets are projected to average 23.3 per cent for the Twelfth Plan period, nearly
the same as in the Eleventh Plan period.
2.52. The overall domestic savings rate is projected to increase from an estimated 30.5 per cent in 2011 12 to 36.3 per
cent in 201617, and average 34.2 per cent for the Twelfth Plan period. This would be slightly higher than the 33.1 per cent
recorded in the Eleventh Plan period. Since the projected aver- age investment rate (GDCF, including errors
and omissions) in the Twelfth Plan (at current prices) is 37.0 per cent and the projected gross domestic sav- ings is 34.1
per cent, the net external financing needed for macroeconomic balance should average 2.9 per cent.
6/23/2014 Evernote shared notebook:
https://www.evernote.com/pub/crazyphoton/interviewprep#st=p&n=7a5f4d76-f67a-42ea-8484-e7aaa3d53c47 21/34
BoP Flows
as the stock of foreign investment builds up in India, the net investment income is increasingly becoming a larger negative
number, going from ()0.6 per cent of GDP in 200405 to ()0.9 per cent in 201112.
Merchandise exports would be over $600 billion. The average of the Twelfth Plan would be 18 per cent.
Merchandise imports are also expected to increase as a proportion of GDP to average about 27 per cent of GDP dur- ing
the Plan period. The merchandise trade deficit would therefore average 9 per cent of GDP.
2.57. The net positive balance on trade in services is expected to increase only slightly to 3.4 per cent of GDP from 3.2
per cent in the Eleventh Plan. Private remittances averaged 3.1 per cent of GDP in the Tenth Plan, which increased to 3.5
per cent in the
The net FDI inflow was thus 1.1 per cent of GDP for the Eleventh Plan as a whole. Portfolio equity inflows fluctuated to a
greater extent, from 1.12.4 per cent of GDP in the Eleventh Plan period, and averaged 1.3 per cent of GDP
Debt inflows together are 1.5%
Total capital inflows thus are 4%.
12th plan assumes FII inflows would slow to 0..5% from 1.3%.
global perceptions about our macroeco- nomic stability are critical for maintaining access to capital flows and, as pointed
out above, the fiscal deficit is a performance parameter of critical impor- tance.
Scenarios of 12th Plan
the second scenario Insufficient Action describes the consequences of half hearted action in which the direction of policy
is endorsed, but sufficient action is not taken. The growth in this scenario declines to around 6 per cent to 6.5 per cent.
The third scenario Policy Logjam, projects the consequences of Policy Inaction persisting too long. The growth rate in
this scenario can drift down to 5 per cent to 5.5 per cent.
Achievements of 11th Plan
Agricultural GDP growth accelerated in the Eleventh Plan, to an average rate of 3.3 per cent, compared with 2.4 per
cent in the Tenth Plan, and 2.5 per cent in the Ninth Plan.
55 GW capacity
GDP growth in the Eleventh Plan 200708 to 201112 was 7.9 per cent compared with 7.6 per cent in the Tenth Plan
poverty line population
unemployment
6/23/2014 Evernote shared notebook:
https://www.evernote.com/pub/crazyphoton/interviewprep#st=p&n=7a5f4d76-f67a-42ea-8484-e7aaa3d53c47 22/34
Eleventh Plan, yielding an annual average net capital inflow of 4.1 per cent of GDP during the Eleventh Plan. Since the
average current account deficit was 2.7 per cent of GDP, the net capital inflows exceeded what was required to finance the
current account deficit and contributed to a build up of forex reserves.
teledensity 80
minor ports did well in 11th plan than major ports because there were some issues regarding incentives for PPP
The rate of growth of real consumption per capita in rural areas in the period 200405 to 201112 was 3.4 per cent per
year which was four times the rate in the previous period 199394 to 200405.
Rural real wages increased 6.8 per cent per year in the Eleventh Plan (200708 to 201112) compared to an average 1.1
per cent per year in the previous decade, led largely by the governments rural policies and initiatives.
achievements in IMR, MMR, TFR, institutional deliveries, immunization.
Environment
Planning Commission has commissioned an Expert Group under Professor Partha Dasgupta to prepare a template for
estimat- ing green national accounts which would measure national production while allowing for negative effects on
national resources.
Eco-efficiency is achieved by the delivery of competi- tively priced goods and services that satisfy human needs and bring
quality of life, while progressively reducing ecological impacts and resource intensity throughout the life-cycle, to a level at
least in line with the earths carrying capacity.
socio-efficiency, that is, the relation between a firms value added and its social impact.
National Clean Energy Fund (NCEF) in 2010 by imposing a cess on coal at an effective rate of !50 per tonne. The
Government expects to collect !10000 crore under the Clean Energy Fund by 2015.
4.13. Compensatory Afforestation Fund is an inno- vative mechanism for attracting additional resources to the forestry
sector. Money is collected for com- pensatory afforestation from user agencies in lieu of the land granted for non-forestry
purpose, presently at the rate of !0.8 million per hectare.
National Gene Fund, has been established, which will be used to build capacity at Panchayat level for in situ conservation
of genetic diversity of indigenous crop varieties.
n 2031, Indias per capita GHG emissions will be under 4 tonnes of Carbon Dioxide equivalent (CO2eq.) which is
lower than the global per capita emission of 4.22 tonnes of COeq. in 2005. current emissions is 1.18 tonnes.
eco-industrial park (EIP) - hub or cluster of industries to improve economic performance while minimizing environmental
impacts. DMIC and SEZs will be converted into EIP.
(SC) power plants, which operate at steam condi- tions 560o C/250 bars, can achieve a heat rate of 2235 kCal/kWh as
against a heat rate of 2450 kCal/ kWh for sub-critical power plants. The specific CO2 emission for super-critical plants is
0.83 kgagainst 0.93 kg/kWh for sub-critical plants. Ultra super critical plants operate at USC steam conditions (620
C/300 bars) and can achieve a much lower heat rate of 1986 kCal/kWh, while the specific CO2 emissions are only 0.74
kg/kWh. An Ultra Super Critical (USC) coal-based power plant has an efficiency of 46 per cent compared with 34 per cent
for a sub critical plant and 40 per cent for a Super Critical (SC) plant.
wind: the global practice is now to build towers in the range of 80120 m, which significantly increases the power
potential. At the same time, the size of wind turbines has increased while the earlier turbines were typically less than 1
MW, the recent designs go up to over 5 MW. Taking these into consideration, the wind potential in India is now estimated
at about 103000 MW for 80 m hub height. carbon capture.
we could safely target a wind capacity addition of 30000 MW by 2020. However, as noted in Chapter 12, wind potential is
unevenly distributed across the country; only Karnataka, Tamil Nadu, Andhra Pradesh, Maharashtra and Gujarat
have significant potential.
Wind power has significant seasonal and even intra-day variations.Wind capacity addition needs to be complemented by
other energy sources, which have a quick ramp-up time. 12th plan talks about national wind energy mission.
6/23/2014 Evernote shared notebook:
https://www.evernote.com/pub/crazyphoton/interviewprep#st=p&n=7a5f4d76-f67a-42ea-8484-e7aaa3d53c47 23/34
Environmental Performance Index (EPI) 7.70. The Planning Commission is in the process of developing an EPI to
incentivise states for environ- mental performance through budgetary allocationsso on. (
Solar
Solar photovoltaic technologies have several advantages: they can provide distributed power, enable quick capacity
addition and work with diffused solar radiation.
Solar thermal technologies are conducive for utility-scale power generation, and have the advantage of energy storage.
However, solar thermal technologies only work on direct beam radiation and utility-scale plants require large amount of
land and water
The NSM has targeted 2000 MW of off-grid solar power by 2022. Current guidelines limit a solar micro-grid to 100 kW per
site and provide a capital subsidy of 30 per cent. The concept of micro-grid, even though attractive, has so far not been
effective in augmenting rural power generation. This is mainly because the developers have found it difficult to
get reasonable returns on their investments as they are unable to collect adequate revenues to cover operating expenses
despite the initial capital subsidy.
4.46. Since the capital subsidy mechanism is not sufficient to incentivise developers to take the risk of setting up micro-
grids, there is a need to examine other options given that rural electricity supply causes loss to the power utilities and it
could take several years before reliable grid power reaches all the villages.
First, there is a need for relaxing the cap on total and site-based project capacity.
Second, there is merit in providing a generation-based incentive, similar to that provided for grid-connected systems
enabling local panchayats with a stake in ownership could ensure local maintenance and operation, as also community-
ownership leading to improved payment collection.
The government should immediately classify solar power projects as priority lending so that banks start giving it
due importance in their credit plans.
Our customs duty structure should not be inverted. Export subsidies (explicit and implicit) available to foreign
manufacturers must be matched by tariff
Cap and trade design leaves out many small and medium organizations (who together may release significant portion of
the emissions). Carbon tax covers the entire economy, including automobiles, households and other units impossible to
reach in a cap-and- trade.
cap and trade: Political pressures could lead to different allocations of allowances, which affect distribution, but not
environmental effectiveness and cost-effectiveness Political pressures could lead to exemptions of sectors and firms,
which reduces environmental effectiveness and drives up costs.
Iron and Steel, Cement, Chemicals and Petrochemicals, Pulp and Paper and Aluminium are the five most energy-intensive
industrial sectors in India
The PAT scheme is an energy intensity type of cap-and-trade scheme as it does not place an abso- lute cap on the total
energy consumption
15 Given the relatively smaller size of the average Indian vehicle, the Indian vehicle fleet is among the most fuel-efficient in
the world.
There has been a tendency for vehicles to get heavier without a corresponding increase in capa- city, as seen in the 2 per
cent per annum increase in average kerb weight of cars sold in India. This is not a desirable trend as it leads to increased
fuel consumption without additional benefits. There- fore, standards must contain an explicit disincen- tive against up-
weighting of vehicles. This can be achieved by making the standards not linear, but a sub-linear function of the vehicle
weight. In the
the energy intensity of rail freight being 0.18 MJ/ tonne-km, while the intensity for road freight being 1.6 MJ / tonne-km ,
that is a nine-fold difference. but share of railways in frieght has declined. hence the need to increase passenger fares.
6/23/2014 Evernote shared notebook:
https://www.evernote.com/pub/crazyphoton/interviewprep#st=p&n=7a5f4d76-f67a-42ea-8484-e7aaa3d53c47 24/34
that is a nine-fold difference. but share of railways in frieght has declined. hence the need to increase passenger fares.
current road freight is inefficient because of reasons such as sub-optimal utilization of trucks, inefficient border crossing,
toll regimes,
center can use the JNNURM funding to meet its objectives.
4.100. Most urban bus utilities in the country are financially unviable, and a significant part of their financial burden is due
to capital expenditure (to buy buses) and taxes. Some studies23 suggest that these expensesincluding various taxes
on fuel form about 20 per cent of the total expenditure of a bus utility, and that these are comparable to or higher than
taxes on private vehicles. Such taxation policy is clearly contrary to the objective of promoting
Lamp Yojana (BLY) provided an innovative business model to sell CFLs to households at the same price as incandescent
bulbs, the balance being recovered as carbon credits. However, a sharp decline in the price of carbon credits has
effectively made this business
Light Emitting Diode (LED), sents an opportunity for another quantum jump in lighting energy efficiency.
ECBC is to become mandatory from 12th plan. Green Building Codes across the country, implementation of these codes
should be made one of the important conditionalties under the revamped JnNURM in the 12th Five Year Plan. The next
Finance Commission should also be given the task of linking financial devolution to urban local bodies to the
implementation of Green Building Codes
National Water Commission is proposed to monitor implementation.
New legal framework for groundwater
- based on public trust doctrine and not absolute dominion. Add to it local bodies principles.
- certain safeguards needed in public trust doctrine.
- social injustice: certain minimum amount guaranteed.
- can panchayats sell the ground water to an industry? farmers may be denied irrigation water in such a case.
Education
About 43 per cent of the public expenditure on education was incurred for elemen- tary education, 25 per cent for
secondary education and the balance 32 per cent for higher education.
mean schooling years is 5.12 < developing countries 7.1 and other BRICS countries.
Primary education
- most important problem in primary - improve learning outcomes.
- another issue is access to disabled children.
- In improving learning outcomes, a major challenge is teachers without required qualifications are appointed. Most such
cases are in Bihar, UP, Jharkhand and W Bengal.
- The capacity, motivation and accountability of teachers to deliver quality education is missing.
- improving school infrastructure inputs as just the starting point in improving educational quality
challenges
- teacher absence, delayed fund flows to schools and administrative capabilities are main challenges. Studies have also
found that improved measurement and management of teacher performance has a significant posi- tive impact on student
learning outcomes.
- 21.38. Clear articulation of learning goals is the criti- cal first step in this process. National learning stand- ards must be
developed on the basis of which States. 1.39. Once basic goals are clearly articulated, all aspects of the elementary
education system (such as methods of teachinglearning, use of materi- als, grouping for effective instruction, optimal
use of time, daily instructional time and number of days of teaching, measurement of progress, capac- ity building and
ongoing support for teachers and administrators) will need to be strongly aligned to the achievement of the learning goals.
- textbooks should be aligned to the overall learning goals
6/23/2014 Evernote shared notebook:
https://www.evernote.com/pub/crazyphoton/interviewprep#st=p&n=7a5f4d76-f67a-42ea-8484-e7aaa3d53c47 25/34
- textbooks should be aligned to the overall learning goals
- Moving From Grade-Level to Ability-Level TeachingLearning 21.50. Recent research in the country and
abroad underlines the need for teaching children from the level that they are and taking them to the level that they need to
be.

- is strong evidence that for children whose home lan- guage is different from the textbook language with no supplemental
parental guidance at home, problems of coping eliminate them from the system. need to develop primers for bridging the
home language to the school language
21.165. Performance-based innovative practices like social audits, linkages with panchayats and munici- palities,
energising and empowering village educa- tion committees, public reporting of expenditures linked to outcomes and
results, and multi-stake- holder dialogues would be used to improve gover- nance in the school system. Most important
would be to empower local communities so that they have better oversight over schools and teachers.
- Parents have to be more actively engaged. Good schools rewarded.
Secondary Education
- 21.90. GERs at the secondary (Class IXX) and senior secondary (Class XI XII) levels are 62.7 per cent and 35.9 per
cent, respectively, leading to a combined GER for Class IXXII at a considerably low 49.3 per
- The Central Government has approved setting up of 2500 Model Schools in PPP mode and a proposal for setting up
3000 ITIs. Recurring tuition support would be provided for up to 1000 students from under privileged categories at par with
the amount that the Central Government spends on a student in Kendriya Vidyalaya. There would be no capital support
and land would have to be procured by the private entity. Infrastructure support shall be made available by the government
for the underprivileged students at the rate of 25 per cent of the recurring tuition support. The concession would be for a
period of 10 years. There will be no financial bidding.
- some States like Rajasthan and MP, the gender gap in GER is as wide as 20 per cent.

- the retention of girls in school remains difficult given that over 63 per cent of rural schools have no usable toilet facilities
for them.10 If the envisaged convergence of the Mahatma Gandhi National Rural Employment Guarantee
Scheme (MGNREGS), Total Sanitation Programme Provision of transport, especially in rural areas, will be made for
schools to avoid school dropout, especially among girls and economically weaker sections due to non-availability of
schools within walking distance.
1.104. Enrolment in more than one-third of the secondary schools in the country is less than 80 students per
school. Secondary and higher secondary schools must be viable and large enough to benefit from investment on quality.
So focus should be on creating composite schools and upgrading existing schools. This will also solve the access issue.
- Focus should be on running schools in multiple shifts, utilising unused land in existing schools.
- It is critical for the country to make second- ary education much more job-relevant through skills training within the
schools. Vocational courses need to be a part of curriculum.
- Private sector - The current licensing and regulatory restrictions in the sector could be eased and a single window
approach should be adopted so that the process of opening new schools by private providers is streamlined.
- Integration of Sports and Physical Education
- all secondary and higher secondary schools would be made to conform to minimum standards in facilities and quality
- Examination Reforms problem solving, analytical skills.
- RMSA should gradually move towards funding States on per child cost basis/norms which would incentivise enrolment,
retention and completion,
- USE OF TECHNOLOGY IN EDUCATION
- rajasthan has large number of teacher vacancies. National professional standards for teachers should be evolved.
6/23/2014 Evernote shared notebook:
https://www.evernote.com/pub/crazyphoton/interviewprep#st=p&n=7a5f4d76-f67a-42ea-8484-e7aaa3d53c47 26/34
Secondary Education: Twelfth Plan Goals
1. Achieve near-universal enrolment in secondary edu-
cation, with the GER exceeding 90 per cent by 2017;
2. Raise the GER at the higher secondary level to 65 per
cent by 2017;
3. Reduce Dropout rate to less than 25 per cent by 2017;
Higher EDUCATION
GER is 18%. Central institutions, which account for 2.6 per cent of the total enrolment; State institutions which
account for 38.5 per cent of enrolment; and private institu- tions that cater to the remaining students. Total 2.5 cr students
enrolled. target is 3.5 cr in 12th plan taking GER to 25%.
- Indian higher education to cross the thresh- old of 15 per cent GER, moving the country from an elite to a mass higher
education system. 50% is universal access.
21.213. Central funding for State higher education is small; its reach is limited, and its impact insignificant. It is poorly
coordinated and plagued by excessive bureaucracy, inefficiencies, low levels of monitoring and poor quality of outcomes. It
therefore, provides little value for money. 12th plan will increase this funding. Central funding for higher education will be
done on a State-specific basis and allocated for the States higher education system as a whole. The UGC would play an
important and more strategic role in allocation and disbursal of Central funds, and thus reform the state education system.
- Currently, for-profit entities are not permitted in higher education and the non-profit or philanthropy- driven institutions are
unable to scale-up enough. Therefore, the not-for-profit status in higher education should, perhaps, be re-examined for
pragmatic. This should, however, be subjected to the necessary oversight and accreditation arrangements to ensure
quality and equity. allowing private institutions to raise funds through public offerings of bonds or shares
- giving priority recognition to the sector, like providing it infrastructure status
Expansion - scaling up capacity in existing institutions. First, large and reputed colleges with necessary capabilities and
diverse learning streams will be converted into full-fledged universities.
Equity - increased budgetary support
Governance - Reduce the role of government in administrative matters. Focus on regulation only. Increase
transparency. affiliating universities will be required to revamp their college development councils and give
greater autonomy to their colleges. some of the large and unwieldy affiliating universities will be bifurcated or trifurcated
into manageable units.ate programmes. some of the large and unwieldy affiliating universities will be bifurcated or
trifurcated into manageable units.
The National Accreditation Regulatory Authority for Higher Educational Institutions Bill that seeks to make accreditation
by independent accredita- tion agencies mandatory for all higher educational. the Education Tribunals Bill to create a
Central tribunal and State-level tribunals for expeditious resolution of disputes relating to insti- tutions, faculty, students
and regulatory authorities; (iv) For eign Educational Institutions (Regulation of Entry and Operations) Bill to enable quality
foreign education institutions to enter and operate in India and regulate operations of foreign education provid- ers; (v)
National Commission for Higher Education and Research (NCHER) Bill to create an umbrella regulatory authority
Recently the Central Government has taken several measures to loosen its grip over institutions funded by it, as in the
case of the Indian Institutes of Management, where the government no longer has any role in the selection of Board
members.
Plan targets increase public funding from 1.22% of GDP to 1.5% (states and center combined). Also to increase fee in
public institutions.
PPP
Resort to PPPs in the social sector often raises concerns about the commercialisation of services that are normally
expected to be provided free or highly subsidised. These are important concerns but they can be addressed by well-
drafted concession agreements and strict monitoring
While extending the concept of PPP to social and urban sector projects, the need for peoples participation in the design
and monitoring of PPP schemes becomes crucial. Local citizens are direct stakeholders in such projects and therefore
6/23/2014 Evernote shared notebook:
https://www.evernote.com/pub/crazyphoton/interviewprep#st=p&n=7a5f4d76-f67a-42ea-8484-e7aaa3d53c47 27/34
and monitoring of PPP schemes becomes crucial. Local citizens are direct stakeholders in such projects and therefore
their support becomes crucial. Therefore, some cities and States have begun to shape PPPs in the social and urban
sectors as PeoplePublicPrivate Partnerships (PPPPs).
Finance
Q. What are the critical issues in the expansion of banking and insurance?
High transaction costs at the bottom of the pyramid. SHGs, microfinance, kisan credit cards, aadhar for KYC.
Universal banking vs narrow banking: it is not clear whether the Indian banking sector has acquired the requisite risk
assessment and project appraisal skills for term loans, without which financing long-duration projects can be hazardous.
assetliability imbalance as deposits are ST and lending become LT. Hence the need of increasing long term savings an
infra debt funds and corporate bonds.
Third, since there has been no change in the sources from which banks can raise their resources, all increases in term
lend- ing are at the cost of funds available for working capital purposes. This leads to smaller and weaker clients being
crowded out from the credit space whenever norms stiffen or investment increases. This makes our banking system less
inclusive than it would otherwise have been.
Q. How to tackle the issue of gold?
closely related to the larger issue of instruments of long-term savingslife insurance, pensions, provident funds and so on.
Inflation indexed bonds
R&D
Instead of all government research funds being allocated to the budget of different scientific departments, there is a case
for creating a new National Research Fund which can receive competing research proposals
PSUs should spend 23 per cent of their sales turnover on R&D contract- ing out research to institutions and
universities.
- The transition rate from X to XI Science is very small as indicated by less than 12 per cent share of students in UG
Science stream. This low enrolment in science stream at higher secondary level and poor-quality education is a constraint
in development of scientific manpower in the country.
- Low levels of funding and segregation of the countrys R&D institutions from universities and colleges have been
responsible for the weak research capacity of Indian universities.
Agriculture
Soil Balance - The way forward is to rejuve- nate the soil and restore soil health through addition of organic matter in large
quantities. Use of organic manures will gradually bring down the dependence on chemical fertilisers. However, the use of
organic manures is discouraged because they receive no sub- sidy while urea is heavily subsidised.
Land acquisition and food security - Concern is often raised in this context about the impact on food security.
This problem is greatly exaggerated because the produc- tivity of land in agriculture at present is very low
New focus has to be on livestock, horticulture, fisheries and rainfed areas.
Key challenges - 1. Increase rice productivity in eastern india and to relieve NW India from this crop because of water
issues. 2. nutrient balance of soil. 3. extend procurement operations to other crops.
Land Reforms
6.8. Legally, land leasing laws in most states either prevent marginal and small farmers from increasing the area they
cultivate by leasing in land,
At the same time, absentee landlordism is high in some regions (especially the hill states and rainfed areas), causing
huge tracts of cultivable fallows to lie idle.
6.9. There is, therefore, a strong case for legalising tenancy and allowing leasing-in and leasing-out land with adequate
safeguards to protect the interests of small and marginal farmers.
states like Punjab and Haryana, large and medium farmers who lease in land from small and marginal farmers invest in
6/23/2014 Evernote shared notebook:
https://www.evernote.com/pub/crazyphoton/interviewprep#st=p&n=7a5f4d76-f67a-42ea-8484-e7aaa3d53c47 28/34
states like Punjab and Haryana, large and medium farmers who lease in land from small and marginal farmers invest in
modern inputs, reap economies of scale and raise farm productivity. The small and marginal farmers who lease out
their higher incomes. In other states like Bihar and Orissa, with low wages and fewer employment opportuni- ties, small
and marginal farmers lease in land, enlarge their holding size and thus afford a reasonable level of living with all attendant
benefits of tenancy like borrowing from financial institutions. The medium and large farmers in these states migrate to
urban areas to take non-farm employment opportunities without any risk of losing their land.
feminisation of agriculture.
Government of Andhra Pradesh, under which poor dalit women formed small groups to buy land collectively for joint
farming, The land was equally divided and registered in the names of individual women. But they are culti- vating jointly by
pooling it. However, experience has shown that government should not purchase land for leasing to D&W farmers, as
attempted in Andhra Pradesh. Government entry in the land market tends to hike up prices, making the scheme
unsustainable.
In 2009 Andhra Pradesh introduced a bill in the Assembly (Self-Help Group Tenancy Bill 2009), which would legally permit
leasing by womens Self- help Groups. Landowners are assured that their titles will not be in jeopardy. However, a flaw in
AP 2009 Bill is that the land will be leased collectively by the group, but can be sub-leased to group members, with the
group bearing liability for the lease. This is retrogressive since default by one member would make the entire group
indebted. Also subleasing will fragment the holdings and undermine potential economies of scale. Also reports indicate
that even the news of this potential legalisation has frozen the land lease market.
6.22. Public Land Banks: Even legal guarantee may be insufficient to mitigate the landowners fear. Enacting a law to
recognise tenancies could freeze the informal land lease market in the short run.
Public Land Bank (PLB) at alise land demand and supply. The PLB would take deposits of land from landowners wanting
to lease out their land, with the surety that they could with- draw their deposit when they wanted. The deposit could be for
one season, one year, or three years and more. On deposit the farmers would get a small pay-ment as incentive, the
amount varying by the period. The landowner would receive an additional fee when the land is leased out. The PLB would
lease out the land under its command to specially designated categories of disad- vantaged farmers, such as marginal
farmers, women, dalits, and tribals, There can be several incentives for farmers to deposit their land in the PLB: (i) a
minimum rent from the PLB even for fallow land; (ii) an addi- tional topping up rent for land that gets leased out; (iii)
development of the land in terms of soil conser- vation and so on, via MGNREGA or other means. with owners being free
to withdraw their land from the Bank with due notice. For the lessees, it would provide D&W farmers access to land for
which they cannot always compete in the open market. The PLB should provide a guaranteed lease and, where possi- ble,
a consolidated plot of reasonable size
current land acquisition under the doctrine of eminent domain and treats people like subjects.
Land Readjustment (LR) is gaining acceptance as an alternative to land acquisition. Under this process, a compact area
is selected in consultation with the land owners for urban expansion/renewal. The municipal authorities provide
infrastructure which is funded by exploiting a part of land. The remaining land, whose value has increased due to provision
of infrastruc- ture, is reallocated back to participating private land- owners. In essence a participatory tool, LR avoids public
discontent and protests to a great extent. It
Land Records
6.58. Once land revenue began to decline in signifi- cance as an element in state income, especially in the 1970s, land
record administration underwent great neglect. even mutation doesnt happen now. multiple departments are involved. we
need a single window system, updated records, land title conclusively proving ownership. National Land Record
Modernisation Programme is on. Torrens Programme may be implemented by the next plan. Critical bottleneck is human
resources.
Nutrition
Poor learning outcomes in our schools are partly because of poor quality of teaching but they are also partly due to high
incidence of child malnutrition, which reduces learning ability.
Food Security Bill
Power
unwillingness on the part of banks to finance power generation projects that are being set up because of doubts that they
will be paid by the dis- coms. A debt restructuring plan, in which State Governments take over a large part of the burden of
pay
6/23/2014 Evernote shared notebook:
https://www.evernote.com/pub/crazyphoton/interviewprep#st=p&n=7a5f4d76-f67a-42ea-8484-e7aaa3d53c47 29/34
Health
At present, less than 30 per cent of outpatient and less than half of inpatient health care capacity of the country is in the
public sector,
Plan will therefore see the transformation of the NRHM into a National Health Mission, covering both rural and urban
areas.
0.45. In order to achieve health goals UHC must build on universal access to services that are deter- minants of health,
such as safe drinking water and sanitation, wholesome nutrition, basic education, safe housing and hygienic environment.
To aim at achieving UHC without ensuring access to the deter- minants of health would be a strategic mistake, and
There is a massive shortage of healthcare professionals
to focus much more on a provision of clean drinking water and sanitation. The incidence of slipped back habitations
appears to be accelerating. Excessive withdrawal of groundwater for irrigation
IT / Mobile based solutions: The parents of babies born in municipal hospitals in Bengaluru get an SMS alert, when the
next vaccination is due.
UHC is a process which will unfold over 2-3 plan periods. expenditure of 2.5% by end of 12th pla,. but it includes 1.5% of
water and sanitation. PPP would be used. RSBY would cover all BPL. Some drugs will be made available universally free
of cost.
NREGA Criticism
Hurting farm profits - If rising wages squeeze farm profitability, the solution lies in raising farm productivity to accommodate
higher wages. We should not be looking to perpetuate a situation where low-cost labour provides the necessary
profit margins for farmers, removing incentives to invest in efficiency improvement.
Bottomless Pit - What is less appreciated is that this has been achieved with a rather modest increase in the share spent
on rural employment schemes out of total Central Plan expenditures. It has increased from an average of 11.8 per cent in
the three years before MGNREGA (200203 to 200405) to 13.3 per cent in the last three (200910 to 201112). This
means that although MGNREGA is not free of leakages, these have declined considerably. Thus, far from opening a
bottomless pit as some critics still claim, the provision of employment as a legal right, has greatly improved the share of
intended beneficiaries\
Q. Why has inflation gone up in India after the crisis?
higher fiscal deficit - caused by both revenue fall and higher subsidies.
global inflation - qe imported since our currency depreciated. and currency depreciated because of bear market +
structural issues dampening foreign investor confidence and higher CAD.
RBI failed to check inflation expectations early and needs to work on credibility side.
But food subsidy in india was 0.8% of GDP in 2004-05 and has remained in the range of 0.75% to 0.9% since then.
Capital gains on assets where STT was charged was as follows: Long term (> 1 year): 0%, Short term 15%.
Now it will be: Long term: 0%, Short term: 50% deduction and then tax @ marginal tax rate.
5. Capital gain on assets where STT was not charged was as follows: Long term (>3 years): 20% with indexation
benefits, Short term: marginal tax rate. Now it will be: marginal tax rate (with indexation benefits) for both short
and long term.
Q. Why has CAD gone up in India after the crisis?
Q. Why is INR one of the weakest currencies in Asia?
1. High CAD, FDI doesn't cover it and so reliance on FII inflows which are volatile.
2. Exports depend on global demand. Imports are inelastic.
III. Problems with Existing Official Poverty Lines

The existing all-India rural and urban official poverty lines were originally defined in
terms of per capita total consumer expenditure (PCTE) at 1973-74 market prices and
adjusted over time and across states for changes in prices keeping unchanged the original
1973-74 rural and urban underlying all-India reference poverty line baskets (PLB) of
goods and services. These all-India rural and urban PLBs were derived for rural and
6/23/2014 Evernote shared notebook:
https://www.evernote.com/pub/crazyphoton/interviewprep#st=p&n=7a5f4d76-f67a-42ea-8484-e7aaa3d53c47 30/34
goods and services. These all-India rural and urban PLBs were derived for rural and
urban areas separately, anchored in the per capita calorie norms of 2400 (rural) and 2100
(urban) per day. However, they covered the consumption of all the goods and services
incorporated in the rural and urban reference poverty line baskets. Three major criticisms
of these poverty lines have been commonly aired. One, the consumption patterns
underlying the rural and urban PLBs remained tied down to those observed more than
three decades ago in 1973-74 and hence had become outdated. Given the rise in the
living standards resulting from accelerated economic growth since the nineteen-eighties,
the consumption pattern of the poor has also been changing but is not reflected in the
poverty lines. Two, crude price adjustment for prices was leading to implausible results
such as proportion of total urban population below poverty line being higher than its rural
counterpart in certain major states. In particular, it was shown by Deaton that Consumer
Price Index for Agricultural Labourers that was used for the rural population understated
the price rise for the rural population and hence understated the extent of rural poverty.
Three, the earlier poverty lines assumed that basic social services of health and education
would be supplied by the state and hence, although private expenditure on education and
health was covered in the base year 1973-74, no account was taken of either the increase
in the proportion of these in total expenditure over time or of their proper representation
in available price indices.
Q. Benefits of NREGA.
Social justice - 51% SC/ST, 47% women. bringing about parity in male - female wages. females participation in nrega is >
female participation in casual labor market in all states.
Benefits poor - benefits female headed households more in terms of income support. eg. in raj, share of nrega earnings in
female headed household income was 15% while male households was 7.5%.
Small and marginal farmers land improved. can tie up with public land bank scheme.
agricultural wages - were growing 0.6% before that. now 6.5% ... a real structural break.
empowerment of panchayats - resources, planning by GS, 50% works executed by GP.
nutrition security + education. people also using the money to start own business. expenditure on transport has fallen.
provides income to poor when they most need it i.e. averse weather conditions. females are more affected by it and hence
a bigger relief.
50% of the S&M farmers whose lands have been worked upon have not returned to nrega.
concept of multiplier and accelerator applied.
people feel the ownership of assets.
climate change and nrega - carbon sequestration potential.
Q. Criticisms of NREGA
reducing availability of labor - minimum wage argument, NSSO data result - in raj, only 14% hh report availability of
alternative work during NREGA days. nrega has increased rural workforce participation rates by drawing many into work
force who were earlier out of job markets. reduction of agricultural work force was a phenomenon which started before
nrega. most of nrega works are generated in lean seasons.
inflation - rights of poor + enhance productivity - infra development, land development = enhanced agricultural productivity.
asset quality.
completion rates low (national avg is 60%) - lack of planning (construction of ponds started before monsoon) + irregular
flow of funds + delays in wage payments + a multi year work is treated as incomplete in the first year. instead it should be
split into annual components + sometimes wages are revised up or material costs rise and new funds not allocated. such
a revision of estimates should be carried out. + No new work should be started if previous works not completed beyond a
threshold.
reduced migration - but nrega has only reduced push migration. pull migration is not affected at all.
Q. inter state variation in female participation - reasons
rajasthan has 70%. southern states have higher. UP bihar, J&K NE states have lower.
limited alternate work opportunities and hence rationing in poorer states.
women facilities.
SHGs, NGOs, govt commitment, cultural acceptance.
Q. The schemes like NREGA is also not functioning? Why?
not right to say not functioning. yes there are problems.
lack of engineers and staff - asset quality. no use of machines - so how can roads be durable? hence the need of more
engineering and planning. ponds are constructed without conceptualization of catchment area. catchment area treatment
is not carried out leading to silting.
lack of staff - work evaluation. in rajasthan, only 10% of participants get paid in 15 days (NSSO)
social audit.
irregular flow of funds and administrative delays.
women participation low in some states.
poor bank coverage, illiteracy.
IT system flaws - not real time.
6/23/2014 Evernote shared notebook:
https://www.evernote.com/pub/crazyphoton/interviewprep#st=p&n=7a5f4d76-f67a-42ea-8484-e7aaa3d53c47 31/34
IT system flaws - not real time.
infrequent meetings and low participation in GS. low awareness.
Q. NREGA 2.0
strengthening social audit.
performance audit by cag. auditors employed at district level.
Q. What about the FDI in the retail sector and organized retail? Do you think it should be there?
yes.
efficiency.
investment - logistics.
Q. Do you think because of recession the capitalism as a system has failed?
no, but emphasized importance of regulations. modern capitalism was anyways far from lassiez fierre.
Q. In the context of the Japanese disaster, do you think we should continue our expansion plans for nuclear energy or
should we stop it?
critical analysis - both safety as well as desirability.
need to make credible plans.
need to have credible institutions - make independent regulator, credible disaster management plans. very often its not
reactors which fail, its human systems which fail. eg. Fukishama. CAG report.
larger question of R&R.
Q. How safe is nuclear energy? Very recently the former chairman of the AEC said that India should not import foreign
reactors. Even he is not so sure about the safety of the rectors.
100% safety can never be there. Science can take us to a reasonable level of safety. But I think more than the safety of
reactors, its the human systems around it which fail. eg. Fukishama. CAG report.
AERB is not independent. In the NSRA, The AEC chairman will also be one of the key members of the Council of Nuclear
Safety that will set the policies with respect to radiation and nuclear safety that will fall under the purview of the NSRA.
the functions of monitoring of radiological exposure as well as the responsibility of radiological surveillance is now with
NPC the operator of the reactors.
there is no overall nuclear and radiation safety policy.
Nor does the AERB have the mandate to take follow-up action with district or state authorities when it detects deficiencies
in emergency preparedness.
The AERB has also not paid any attention to planning for decommissioning nuclear reactors.
Q. How are some of these concerns addressed in nuclear safety bill?
Q. Details of nuclear liability bill
1. The modifications also cap the liability of the operator at $300 mm for unlimited period.
2. The operator can claim the min (damages paid by him, value of the supplier's contract) from the
supplier.
3. The recourse period is max (period specified in the contract, period of initial license which is 5
years).
4. The operator can claim only what is directly paid by him as compensation to the victims whereas the
cost of nuclear damage is much more.
Q. Discom reforms
3. Under the new package, discoms will have to mandatorily revise their tariffs every 9 months. They
will have to issue bonds for half of the accumulated losses and government will guarantee the other
half.
4. A National Electricity Fund has been setup to give interest subsidies (currently $1.6 bio) to discoms.
5. Discoms can be improved by including modern technology and management systems or by
privatization or hiring a management company (system called franchising).
6. Open Access Policy (where consumers are free to chose from competing discoms) should be followed.
Q. Direct Cash Transfer.. Pros and Cons. Your view?
big step - corruption.
cons - no banking infrastructure, girls and children neglected, price fluctuations.
Q. How will u improve agriculture? short term solutions
inefficient supply chain - encourage direct marketing and e-marketing solutions, build additional storages.
more farm credit - kisan credit cards, gender imbalance.
easier availability of inputs.
Q. What are the main problems with indian economy these days?
6/23/2014 Evernote shared notebook:
https://www.evernote.com/pub/crazyphoton/interviewprep#st=p&n=7a5f4d76-f67a-42ea-8484-e7aaa3d53c47 32/34
Q. What are the main problems with indian economy these days?
growth - structural reforms, human resource development
inflation - CAD control, structural supply side reforms, RBI credibility
CAD - oil, gold
inequality - inclusive growth
crony capitalism
Q. Labour laws do you favor them? should hire and fire be the policy?
90% economy already working on hire and fire. but labor rights should be safeguarded.
advantages of hire and fire - contract, capital intensive, economic efficiency.
unique PF account to each labor and direct credit in PF account instead of giving to contractor.
Q. why schemes are failing at grassroot level?
monitoring - social audit, IT systems.
local people involvement.
Q. What is Knowledge Economy ?Are only Services covered under Knowledge Economy?
Q. Having high Share of Service Sector in GDP is good or Bad for India ?
Limited domestic demand argument vs export demand argument: ToT worsen... but not in India. Plus
trade in services is currently very low and is only bound to increase.
Self feeding growth argument: Banga and Goldar (2004) find a positive relationship between use of services input and
industrial productivity.
Law of diminishing returns argument: As economies become more service oriented, their growth slows. but Indian
productivity is high and growing at much higher rate - comparable to China. potential for new innovations is still great in
India.
outlook of sustainability of growth in modern, hybrid and traditional services - modern has grown so far.. still a lot of scope
left in hybrid and traditional.
Q. machines creates unemployment. So how u will tackle this problem.
encourage labor intensive sector.
improve supply chain of handicrafts.
Q. If you were to tell the 1 biggest challenge that the country faces today, where we should focus, what would it be?
governance - we need to cut down red tape, start taking decisions, rule of law, increase transparency.
Q. Do you know about Gandhian economics? Do you think Gandhian economics is relevant today?
rural urban migration.
empowerment, inequality.
Labor intensive - employment.
Q. What in your opinion are the three big problems of India? How can they be tackled?
poverty, unemployment, education, governance.
Q. What are the main problems of agriculture today?
Marketing.
Storage.
Rain fed areas - 60% of area, pulses, horticulture.
Lack of investment.
Unorganized nature / small OHLs.
Q. On one hand we want people to move out from agriculture into services and industry. On the other hand we want to
increase agriculture productivity. Isn't there an inherent contradiction here?
Talk about wage goods gap - inflation.
Talk about agriculture providing market to industry and services.
Talk about push migration.
Talk about improving lives of agriculture dependent households.
Q. What can be done to increase growth of agriculture?
Growth to come from livestock, horticulture and fisheries (give stats) because of shift in demand patterns. We must shift
our efforts accordingly. Need better post harvest management. Livestock growth is more inclusive. If markets are better i.e.
supply chain efficient, then the farmers will get better remuneration and hence incentives to invest more. Food processing
industry covers only 2-3% of fruit and vegetable production in India (as against 30% in Thailand and 70% in Brazil). So
farmers are reluctant to follow price signals. Total storage capacity: 110 MT.
Focus on rain fed areas - 60% of area + pulses + horticulture. Climate change will affect them more. Need to make
agriculture drought proof. Investment in R&D. ICAR spends only 13% of its funds in rainfed area research.
Increased investment - both public and private.
Q. What were the problems of APMCs?
In most states APMCs don't function - state govts supersede them and managed by bureaucrats.
The commissions collected never ploughed back to develop infrastructure and insteed became a revenue source for govt.
Hence markets remained poor.
Traders formed strong unions and prevented competition and exploited peasants.
Q. Provisions of model act?
6/23/2014 Evernote shared notebook:
https://www.evernote.com/pub/crazyphoton/interviewprep#st=p&n=7a5f4d76-f67a-42ea-8484-e7aaa3d53c47 33/34
Q. Provisions of model act?
Direct marketing, contract farming, private markets, farmers' markets allowed.
single point of levy, abolition of commission agents, use of funds for market development only.
Q. What are the needs in irrigation?
Irrigation reforms to increase PPP in irrigation. example of water user associations. Viability gap funding in agro markets,
fertilizers, storage, soil testing labs. Gap between potential created and utilized exists in all states. One reason for this
is that irrigation potential is calculated on the basis of the volume of water expected in the reservoir divided by a presumed
depth of irrigation required for a presumed cropping pattern. However, the total water available is often less than assumed
due to faulty project designs, faster siltation etc. There is also a widespread tendency for those near the headworks to
appropriate much larger amount of water shifting to water-intensive crops leaving less water for tail-enders. All these
developments are encouraged by lack of co-ordination across agencies and departments, and the inadequate or
complete absence of involvement of water users through Water User Associations (WUAs). Irrigation efficiency is currently
35% for large projects.
Q. Water - a paradigm shift - 12th plan approach.
Irrigation - state irrigation departments - farmer linkage broken. restore it by way of matching grant. Strengthen WUAs.
Command area development should be an integral part of the project itself and not left for later.
Ground water - from absolute dominion to public trust doctrine. But concerns should be kept in mind. separation of
feeders.
water pricing - differential pricing.
water supply - reduce pipeline distance. focus on local water bodies. sewage should be incorporated in water projects
itself. (Tropical enteropathy is the link between sanitation and malnutrition.)
watershed management - MGNREGS can be turned into the biggest watershed management scheme.
water footprints and audit for industry.
national water framework law - a set of general principles according to which legislative and executive powers can be
exercised.
Q. What is India's seed patent system?
Sui generis system - farmers develop seeds to adapt to local conditions. Their contribution recognized - they can develop
and sell their own seeds (but not branded). Also if MNCs develop new seeds using local varieties and knowledge, the local
communities will be acknowledged as contributors and benefits need to be shared.
Q. What are the features of Seeds Bill, 2004?
Any seed sold will have to be registered and required to meet certain minimum prescribed standards. Farmers can claim
compensation on non performance. Farmers' seeds exempted.
Q. What is seed replacement rate?
% area sown by certified seeds to total area sown under the crop.
Q. What are the issues in fertilizers?
Domestic production needs to be more - Urea regime should be changed.
Vast regional inequalities in consumption - Punjab vs NE, Rajasthan.
Soil nutrient balance - nutrient based subsidy regime + organic fertilizers. In 1992, government freed up P& K fertilizers.
This led to a sharp increase in their price and a deterioration in the nutrient balance (from 4:2:1 recommended to 10:3:1 in
1996- 97). Subsequently ad hoc subsidies were given on P&K as well to restore nutrient balance. Then in 2011, nutrient
based subsidy scheme was started which is applicable to P & K fertilizers.
Q. How can cash transfer help us in agriculture?
We need 'smart' subsidies - subsidies which are better targeted, differential to keep local variations in
mind (rain fed vs irrigation, small farms vs large farms), better delivery vehicles (like debit input cards
where the farmer is allotted a total amount and he can chose the composition of subsidies himself
which will also stop overuse of vital resources like water).
Q. What are the modern land reforms which are needed?
Amendment to tenancy reforms - protect ownership of land while building adequate safeguards for S&M farmers. Need
innovative solutions - public land banks example. We also need it to promote investment in farms and investments are
becoming more and more critical now. If there is an investment to be made, the tenant will not invest because of the
informal nature of his tenancy. To overcome this we need long term tenancies which are only possible if we formalize
tenancies.
Women empowerment - female headed households. need to give them titles and credit.
Q. What can we do to reduce farm suicides?
Need to derisk the farmer. make MSPs more effective. making sure appropriate seeds are available and crop varieties suit
local conditions. better post harvest management and loans against storage. insurance products.
Rashtriya Krishi Vikas Yojana
Increase investment in agriculture - 4 bio scheme. BGREI, NFSM, Saffron mission are all part of it. soil testing
laboratories.
incentivises states to increase public investment. projects satisfying criteria are approved.
BGREI - cluster approach, block demonstrations, mechanisation, input availability.
NFSM - 10 MT rice, 8 MT wheat, 2 MT pulses. focus on district where yield below state average (rice and wheat). for
6/23/2014 Evernote shared notebook:
https://www.evernote.com/pub/crazyphoton/interviewprep#st=p&n=7a5f4d76-f67a-42ea-8484-e7aaa3d53c47 34/34
NFSM - 10 MT rice, 8 MT wheat, 2 MT pulses. focus on district where yield below state average (rice and wheat). for
pulses focus was where area and productivity enhancement potential was there. problems - in some state, SRRs were
targeted without corresponding seed production increase plans.
NFSM - 2 : Now includes coarse crops as well and target for increase is 25 MT.
Q. What do you know about international trade in agriculture?
Indian exports were $24 bio whereas imports were $11 bio giving a surplus of $13 bio. Cotton, marine
products, oil meals, basmati rice and sugar are main exports. Main imports are vegetable oils, wood
and pulses.

Potrebbero piacerti anche